National Insurance Company Limited Assistant Examination-2015 Online Examination Held on April 26, 2016 Question Paper With Answer Key

National Insurance Company Limited Assistant Examination-2015 Online Examination Held on April 26, 2016
National Insurance Company Limited Assistant Examination-2015 Online Examination Held on April 26, 2016 Question Paper With Answer Key

National Insurance Company Limited Assistant Examination-2015 Online Examination Held on April 26, 2016

Part I Reasoning

Directions (Q. Nos. 1-5) In these questions, two statements followed by two conclusions numbered I and II have been given. You have to take the given statements to be true even if they seem to be at variance from commonly known facts, and then decide which of the given conclusion(s) logically follows from the given statements disregarding commonly known facts.

Give answer

(a) if only conclusion I is true

(b) if neither conclusion I nor II is true

(c) if both conclusions are true

(d) if only conclusion II is true

(e) if either conclusion I or II is true

1. Statements Some jars are bowls.

Some spoons are jars.

Conclusions I. No bowl is a spoon.

II. Some jars are definitely not bowls.

Answer: (b)

2. Statements Some paints are brushes.

No brush is a canvas.

Conclusions I. No paint is canvas.

II. All brushes are paints.

Answer: (b)

3. Statements All roots are flowers.

No root is a thorn.

Conclusions I. No thorn is a flower.

II. All flowers are thorns.

Answer: (b)

4. Statements All watches are clocks.

Some watches are towers.

Conclusions I. At least some clocks are towers.

II. Some towers are definitely not clocks.

Answer: (a)

5. Statements All buses are trucks.

All cars are buses.

Conclusions I. All cars are trucks.

II. All buses are cars.

Answer: (a)

Directions (Q. Nos. 6-10) Study the following arrangement carefully and answer the given questions :

R # T U 3 D $ J @ B E © W 1 A F % P 2 4 Q I N 6 M ⋆ Z 5

6. If all the numbers in the given arrangement are dropped, which of the following will be ninth from t he right end?

(a)  A

(b)  F

(c)  B

(d)  P

(e)  None of these

Answer: (b)

7. Which of the following will be the sixth to the left of the fourteenth from the left end of the given arrangement?

(a)  J

(b)  Q

(c)  N

(d)  D

(e)  None of these

Answer: (a)

8. Which of the following is the fifth to the right of the thirteenth to the left of Q in the given arrangement?

(a)  W

(b)  @

(c)  U

(d)  ⋆

(e)  None of these

Answer: (a)

9. How many such vowels are there in the given arrangement,, each of which is immediately preceded by a consonant and immediately followed by a number?

(a)  None

(b)  One

(c)  Two

(d)  Three

(e)  Four

Answer: (c)

10. Four of the following five are alike in a certain way based on their positions in the given arrangement and thus form a group. Which is the one that does not belong to that group?

(a)  B @ 9

(b)  P % 4

(c)  N I M

(d)  3 T $

(e)  A 1 %

Answer: (d)

11. How many such pairs of letters are there in the word OVERSEE each of which has as m any letters between them in the word (in both forward and backward directions) as they have between them in the English alphabetical series?

(a)  Three

(b)  One

(c)  Two

(d)  None

(e)  More than three

Answer: (a)

Directions (Q. Nos. 12-16) In these questions, relationship between different elements is shown in the statements. The statements are followed by conclusions. Study the conclusions based on the given statement and select the appropriate answer.

Give answer

(a) if only conclusion I is true

(b) if neither conclusion I nor II is true

(c) if both conclusions are true

(d) if only conclusion II is true

(e) if either conclusion I or II is true

12. Statement S ≤ T = E ≤ P = N≤ G

Conclusions I. G > S        II. G = S

Answer: (b)

13. Statement B > R ≥ R A = N < D ≤ S

Conclusions I. B > N       II. S ≥A

Answer: (a)

14. Statement T ≥ I < G ≤ E = R < S

Conclusions I. R > T        II. S > I

Answer: (d)

15. Statements U > B ≥ M ≤ G; L < Y ≤ M

Conclusions I. U > L II. G ≤ L

Answer: (a)

16. Statement P < R ≤ A ≤ I > S ≥ E

Conclusions I. P < I                   II. E ≤ A

Answer: (a)

Directions (Q. Nos. 17-21) Study the following information carefully and answer the given questions.

A, B, C, D, E, F, G and H are sitting around a circular table with equal distances between each other, facing the centre, but not necessarily in the same order. D sits third to the right of E. A sits second to the right of D. H sits to the immediate left of C. F sits second to the left of H. G is an  immediate neighbour of E.

17. Which of the following is true with respect to B as per the given arrangement?

(a)  All the given statements are true.

(b)  Only one person sits between B and A.

(c)  Only two people sit between B and G.

(d)  Only three people sit between B and H.

(e)  F sits third to the left of B.

Answer: (d)

18. Who amongst the following represents the immediate neighbours of D?

(a)  H, F

(b)  B, G

(c)  B, F

(d)  F, G

(e)  C, F

Answer: (c)

19. As per the given arrangement, four of the following five are alike and thus form a group. Which of the following does not belong to that group?

(a)  DE

(b)  CF

(c)  FG

(d)  HD

(e)  AC

Answer: (e)

20. If all the persons are made to sit in alphabetical order in anti-clockwise direction, starting from A, the positions of how many (excluding A), would remain unchanged?

(a)  Four

(b)  One

(c)  Three

(d)  None

(e)  Two

Answer: (b)

21. Who amongst the following sits third to the left of C?

(a)  E

(b)  D

(c)  B

(d)  G

(e)  F

Answer: (e)

22. Among five people J, K, L, M and N (each running at a different speed), who ran at the third fastest speed?

M run faster than L, but slower than K. J ran slower than only one person. L did not run at the slowest speed.

(a)  Cannot be determined

(b)  M

(c)  N

(d)  L

(e)  K

Answer: (b)

23. T is married to N. R is the sister of N. P is the father of R. J is married to P. How is T related to J?

(a)  Brother

(b)  Sister-in-law

(c)  Son-in-law

(d)  Son

(e)  Cannot be determined

Answer: (e)

Directions (Q. Nos. 24-28) Study the information and answer the given questions.

‘sudden change is visible’ is written as ‘bo st tz rk’

‘visible change in approach’ is written as ‘xu tz yi bo’

‘change is good also’ is written as ‘cd rk bo en’

‘want good approach now’ written as ‘pm qs yi cd’

(All the codes are two-letter codes only)

24. What is the code for ‘sudden’ in the given code language?

(a)  bo

(b)  rk

(c)  st

(d)  xu

(e)  lz

Answer: (c)

25. Which of the following may represents code for ‘visible in general’ in the given code language?

(a)  lz yi jr

(b)  qs tz xu

(c)  cd xu pm

(d)  xu bo tz

(e)  xu ws tz

Answer: (e)

26. Which of the following represents the code for ‘approach is’ in the given code language?

(a)  xu yi

(b)  pm yi

(c)  rk tz

(d)  rk yi

(e)  pm tz

Answer: (d)

27. What does the code ‘qs’ stand for in the given code language?

(a)  is

(b)  good

(c)  Either ‘now’ or ‘want’

(d)  Either ‘want’ or ‘good’

(e)  approach

Answer: (c)

28. What is the code for ‘good’ in the given code language?

(a)  en

(b)  cd

(c)  pm

(d)  rk

(e)  yi

Answer: (b)

Directions (Q. Nos. 29-33) Study the following information and answer the given questions :

Eight people (A, B, C, D, E, F, G and H) live on eight different floors of a building not necessarily in the same order. The lowermost floor of the building is numbered 1, the one above that is numbered 2 and so on till the topmost floor is numbered 8.

A lives on one of the floors above D. Only four people live between A and D. E lives immediately above D’s floor. H lives on an odd numbered floor below E. Only three people live between H and G. C lives immediately below B’s floor.

29. Four of the following five are alike in a certain way based on the given arrangement and hence form a group. Which of the following does not belong to that group?

(a)  GB

(b)  EH

(c)  DC

(d)  FG

(e)  BE

Answer: (a)

30. Which of the following statements is true, as per the given arrangement?

(a)  Only three people live below C’s floor.

(b)  B lives on the floor numbered 7

(c)  None of the given statements is true.

(d)  Only two people live between E and F.

(e)  H lives on the floor numbered 3.

Answer: (b)

31. D lives on which of the following floor numbers?

(a)  Floor number 8

(b)  Floor number 6

(c)  Floor number 7

(d)  Floor number 1

(e)  Floor number 3

Answer: (e)

32. Who amongst the following lives on floor number 2?

(a)  F

(b)  B

(c)  D

(d)  C

(e)  G

Answer: (a)

33. Who amongst the following lives immediately above E’s floor?

(a)  G

(b)  D

(c)  A

(d)  F

(e)  C

Answer: (a)

Directions (Q. Nos. 34-35) Read the following information carefully and answer the given questions :

Point C is 12 m to the West of point D. Point F is 8 m to the North of point D. Point Y is 5 m to the East of point F. Point Y is 14 m to the North of point N. Point N is 5 m to the East of point S.

34. Towards which direction is point C with respect to point N?

(a)  South-West

(b)  South

(c)  North-West

(d)  South-East

(e)  North-East

Answer: (c)

35. Which of the following points is/are exactly 6 m away from point S?

(a)  C

(b)  D

(c)  Y

(d)  F

(e)  C and F

Answer: (b)

Directions (Q. Nos. 36-40) The following questions are based on five words given below :

            URN DEN  MAT FOR  SKI

(The new words formed after performing the mentioned operations may or may not necessarily be meaningful words.)

36. If the positions of the first and the second alphabet in each word is interchanged, which of the following will from a meaningful English word?

(a)  SKI and MAT

(b)  URN

(c)  DEN

(d)  FOR

(e)  URN and MAT

Answer: (b)

37. If the given words are arranged in the order as they would appear in dictionary from left to right, which of the following will be second from the right end?

(a)  SKI

(b)  FOR

(c)  DEN

(d)  MAT

(e)  URN

Answer: (a)

38. If in each of the given words, each of the consonants is changed to previous letter and each vowel is changed to next letter in the English alphabetical series, in how many words thus formed will an alphabet appear twice?

(a)  Three

(b)  None

(c)  More than three

(d)  One

(e)  Two

Answer: (d)

39. How many letters are there in the English alphabetical series between the first letter of the word which is second from the right and first letter of the word which is second from the left of the given words?

(a)  Three

(b)  One

(c)  None

(d)  Two

(e)  Five

Answer: (b)

40. If the second alphabet in each of the words is changed to next alphabet in the English alphabetical order, how many words will be formed in which two or more vowels appear (same or different vowels)?

(a)  Four

(b)  Two

(c)  Three

(d)  One

(e)  None

Answer: (e)

Part II English Language

Directions (Q. Nos. 41-45) In each of the following questions, a sentence with four words is bold type is given. One of these four words may be either wrongly spelt or inappropriate in the context of the sentence. Find out the word which wrongly spelt or inappropriate, if any. That word is your answer. If all the words given bold are correctly spelt and also appropriate in the context of sentence, mark ‘All correct’ as your answer.

41. The guessed took undue advantage of our hospitality and refused to leave.

(a)  guessed

(b)  undue

(c)  advantage

(d)  hospitality

(e)  All correct

Answer: (a)

42. As days passed by, the health of the old man gradually weakened.

(a)  passed

(b)  health

(c)  gradually

(d)  weakened

(e)  All correct

Answer: (d)

43. On seeing the ripe fruit kept in the bowl he could not resist the urge take a byte out of it.

(a)  bowl

(b)  resist

(c)  urge

(d)  byte

(e)  All correct

Answer: (d)

44. He vondered how the lizard had managed to survive behind the wall for so many years.

(a)  vondered

(b)  lizard

(c)  managed

(d)  survive

(e)  All correct

Answer: (a)

45. It was a crowded flight and one of its passengers was a beautiful lay who had bordered with a lot of luggage.

(a)  crowded

(b)  flight

(c)  passengers

(d)  bordered

(e)  All correct

Answer: (d)

Directions (Q. Nos. 46-50) Rearrange the following five sentences A, B, C, D and E in the proper sequence to form a meaningful paragraph, then answer the given questions.

(A) It was always breaking its string, sitting down on the tops of houses, getting stuck in trees and refusing to rise higher than a yard from the ground.

(B) As a result, they were obliged to fly day and night for a living and were never able to give any time to their children or to bring them up properly.

(C) Perhaps, they were very poor people, just made of newspaper and little bits of common string knotted together.

(D) I have often sat and thought about this behaviour of the kite and wondered who its father and mother.

(E) It was the most tiresome kite in the world.

46. Which of the following should be the FOURTH sentence after rearrangement?

(a)  A

(b)  B

(c)  C

(d)  E

(e)  D

Answer: (c)

47. Which of the following should be the SECOND sentence after rearrangement?

(a)  A

(b)  B

(c)  C

(d)  D

(e)  E

Answer: (a)

48. Which of the following should be the LAST (FIFTH) sentence after rearrangement?

(a)  A

(b)  B

(c)  C

(d)  E

(e)  D

Answer: (b)

49. Which of the following should be the THIRD sentence after rearrangement?

(a)  A

(b)  B

(c)  C

(d)  D

(e)  E

Answer: (d)

50. Which of the following should be the FIRST sentence after rearrangement?

(a)  A

(b)  B

(c)  C

(d)  E

(e)  D

Answer: (d)

Directions (Q. Nos. 51-60) Read the following passage carefully and answer the questions given below it. Certain words are given in bold to help you locate them while answering some of the questions.

King Harish loved his people and look after the affairs of the kingdom well. One day he and his Minister Chandan took a stroll through the market. People were buying and selling and there were no beggars to be seen anywhere. The King was delighted to see the prosperity of his kingdom. He turned to Chandan and said, “I want to check firsthand how content my  people are. Summon people from all walks of life to court.” The next day, the King arrived at court and said, “As you King I want to know if all of you are happy. Do you have enough for your needs?” The citizens looked at each other, thought and one-by-one came forward to say that their kitchens have enough food, their trade was going well, their wells were overflowing and the King and kept them safe.

The King was pleased at this but Chandan had frown and he whispered something to the King. The King was astonished but seeing Chandan was serious he turned to the court and made an announcement, “I am delighted you are all happy.

Tomorrow I want all the happy people to gather at the gate of the royal garden. You have to enter the garden from the main gate, walk across and meet me by the gate at the rear of the garden. Each of you will be given a sack and you can pick whatever your heart desires.” The crowd was excited as no one was usually allowed access to the King’s garden which was said to be filled with all kinds of beautiful and strange plants.

The next day, everyone gathered at the gate of the palace garden well before time. At the appointed time, the guards opened the gates and handed out sacks. Citizens began roaming around the garden and filled their sacks with the juicy apples, pomegranates, grapes and mangoes hanging from trees. But as they walked further into the garden they saw trees laden with gold and silver fruits.

They began madly filling their sacks with these precious fruits. Everyone forgot that they had enough for their needs at home and the fruits they had picked earlier were thrown on the ground-forgotten and left to rot. Then with their sacks filled to the top the citizens made their way to the rear gate but they found a rushing stream blocking their path. The current was strong and as there were no boats, the only way to cross was to swim across.

But how could they swim with laden sacks. All stood by the stream except one young men who simply abandoned his sack and swam across. Angry and unhappy the others refused to cross. The King was sad and said, “Yesterday all of you said you were happy but today you are distressed. “Turning to the young man who was smiling he asked, “Tell me why are you not sad?” “Sir, I picked some tasty fruits for my precious daughter but when I saw no other way across. I did not think twice about leaving these behind I am happy you let us wander around in your garden.”

51. Choose the word which is most nearly the opposite in meaning to the word ‘Serious’ given in bold as used in the passage.

(a)  Minor

(b)  Unimportant

(c)  Joking

(d)  Acute

(e)  Slight

Answer: (c)

52. What was Chandan’s reaction to the people’s claim that they were happy?

(a)  He suspected it wasn’t true and wanted to prove it to everyone.

(b)  He was thrilled and urged the King to reward the people.

(c)  He was jealous of their happiness and poisoned the King’s mind against them.

(d)  He knew they were lying and had them punished severely.

(e)  Other than those given as options

Answer: (a)

53. Choose the word is most nearly the same in meaning to the word ‘Well’ given in bold as used in the passage.

(a)  Much

(b)  Healthy

(c)  Glowing

(d)  Fine

(e)  Thriving

Answer: (d)

54. Why were people happy to hear they could spend a day in the palace garden?

(a)  They were keen to spend the day with the King.

(b)  They were starving and would get the chance to get exquisite fruits.

(c)  It was an opportunity to rest and spend time with each other.

(d)  They were curious to see the garden.

(e)  Not clearly mentioned in the passage

Answer: (d)

55. Which of the following can be said about the young man?

(A) He cared for his daughter.

(B) He was happy with his lot in life.

(C) He insulted and made a fool of the King.

(a)  A and B

(b)  Only A

(c)  Only B

(d)  B and C

(e)  All of these

Answer: (c)

56. Which of the following is true in the context of the passage?

(a)  The King distrusted his subjects as they were dishonest.

(b)  Chandan was more powerful and wiser than the King.

(c)  The kingdom was prosperous because the King banished poor people.

(d)  The King trusted Chandan and took his counsel.

(e)  None of the given statements is true in the context of the passage.

Answer: (d)

57. Choose the word which is most nearly the same in meaning to the word ‘Serious’ given in bold as used in the passage.

(a)  Intensely

(b)  Incredibly

(c)  Wildly

(d)  Slowly

(e)  Laughingly

Answer: (b)

58. Choose the word which is most nearly the opposite in meaning to the word ‘Strange’ given in bold as used in the passage.

(a)  Remarkable

(b)  Unseen

(c)  Exceptional

(d)  Artificial

(e)  Familiar

Answer: (e)

59. Why did people throw away the bags of fresh fruit they had picked?

(a)  They realized that the King had tricked them.

(b)  They found that these were too heavy to take across the river.

(c)  They realized that the King may be angry if he saw how greedy they were.

(d)  They discovered that the fruit looked good but was rotten on the inside.

(e)  They were tempted by something more valuable.

Answer: (e)

60. Which of the following can be the lesson of the story?

(a)  Intelligence is the greatest virtue.

(b)  True happiness comes from within not from possessions.

(c)  Think before you speak your mind, others may get into trouble.

(d)  Be ambitious and aim for the best things in life.

(e)  Nothing is given for free, everything has a cost.

Answer: (b)

Directions (Q. Nos. 61-65) Which of the following five words should fill the blanks in the sentences to make them grammatically and meaningfully correct?

61. We don’t want to make your journey …………. by making you sit next to someone with whom you are not comfortable.

(a)  successful

(b)  unpleasant

(c)  favourite

(d)  worthy

(e)  tired

Answer: (b)

62. All the children were …………… as the school had announced an unexpected holiday.

(a)  lone

(b)  ecstatic

(c)  joy

(d)  danced

(e)  requesting

Answer: (b)

63. Since Momo’s hunger could not be satisfied, he ………… to eat, everything around.

(a)  begins

(b)  started

(c)  commence

(d)  want

(e)  make

Answer: (b)

64. The boy felt ………….. after long day’s work.

(a)  tire

(b)  ached

(c)  exhausted

(d)  sleeping

(e)  home

Answer: (c)

65. One day, ……….. serving dinner, the old man whose hands and legs shivered, split food on the table.

(a)  since

(b)  bow

(c)  while

(d)  then

(e)  for

Answer: (c)

Directions (Q. Nos. 66-70) Read each sentence to find out whether there is any grammatical mistake/error in it. The error, if any, will be in one part of the sentence. Mark the part with the error as your answer. If there is no error, mark ‘No error’ as your answer. Ignore the errors of punctuation, if any.

66. A fox watched a/ wild board was sharpened / his tusks on the /trunk of a huge tree.

(a)  A fox watched a

(b)  wild board was sharpened

(c)  his tusks on the

(d)  trunk of a huge tree

(e)  No error

Answer: (b)

67. A fisherman had / been fishing for/ a long time but / without luckily.

(a)  A fisherman had

(b)  been fishing for

(c)  a long time but

(d)  without luckily

(e)  No error

Answer: (d)

68. The lazy grasshopper laughed / at the little aunt / as she was / busy gathering food.

(a)  The lazy grasshopper laughed

(b)  at the little aunt

(c)  as she was

(d)  busy gathering food

(e)  No error

Answer: (d)

69. In the olden days, the camel / was considered / the much handsome animals/ in the jungle.

(a)  In the olden days, the camel

(b)  was considered

(c)  the much handsome animals

(d)  in the jungle

(e)  No error

Answer: (c)

70. The Oak tree always / thought that it was / much stronger than / any other trees in the garden.

(a)  The Oak tree always

(b)  thought that it was

(c)  much stronger than

(d)  any other trees in the garden

(e)  No error

Answer: (c)

Directions (Q. Nos. 71-80) In the given passage, there are blanks, each of which has been numbered. Against each number, five words are suggested, one of which fits the blank appropriately. Find the appropriate word in each case.

One the Wind and the Sun were quarrelling (71) a petty issue. Both of them (72) to be stronger (73) the other. At last they agreed to have a trial of strength.

“Here comes a traveler. Let us see who can strip him of his cloak,” said the Sun. The Wind agreed and (74) to take the first turn. He blew in the hardest possible way. As a (75), the traveler wrapped his cloak even more (76) around him. Then came the Sun, at first he shone very (77) so, the traveller loosened his cloak from his neck. But then h e went on shining brighter and brighter. The traveler started (78) hot. Soon he took off his cloak and put it in his bag. The Wind had to (79) his (80).

71.

(a)  over

(b)  in

(c)  above

(d)  off

(e)  across

Answer: (a)

72.

(a)  said

(b)  claimed

(c)  desire

(d)  privileged

(e)  state

Answer: (b)

73.

(a)  then

(b)  more

(c)  than

(d)  by

(e)  them

Answer: (c)

74.

(a)  picking

(b)  select

(c)  chose

(d)  resolve

(e)  wiling

Answer: (e)

75.

(a)  effect

(b)  circumstance

(c)  conclusion

(d)  result

(e)  return

Answer: (d)

76.

(a)  staff

(b)  deeply

(c)  dim

(d)  heavy

(e)  softer

Answer: (e)

77.

(a)  lighter

(b)  mildly

(c)  dim

(d)  heavy

(e)  softer

Answer: (b)

78.

(a)  experiencing

(b)  loving

(c)  enjoyed

(d)  senses

(e)  feeling

Answer: (e)

79.

(a)  accept

(b)  ask

(c)  give

(d)  conquer

(e)  secure

Answer: (a)

80.

(a)  give

(b)  belief

(c)  defeat

(d)  launch

(e)  include

Answer: (c)

Part III General Awareness

81. The alphabet ‘C’ in the abbreviation CRR stands for

(a)  Cash

(b)  Currency

(c)  Credit

(d)  Commercial

(e)  Capital

Answer: (a)

82. The Minister for ‘Food Processing Industries’ in the Union Cabinet, is

(a)  Thawar Chand Gehlot

(b)  Harsimrat Kaur

(c)  Anant Geete

(d)  Radha Mohan Singh

(e)  Dr. Harsh Vardhan

Answer: (b)

83. Which of the following Indian universities has a unique tradition of having the incumbent Prime Minister as its Chancellor?

(a)  Vishwa Bharati University

(b)  Kuruskhetra University

(c)  Banaras Hindu University

(d)  Jawahar Lal Nehru University

(e)  Baba Saheb Ambedkar University

Answer: (a)

84. Who amongst the following holds the majority stake in the share capital of the recently opened ‘Bharatiya Mahila Bank’?

(a)  Life Insurance Corporation of India

(b)  Insurance Regulatory and Development Authority of India

(c)  Government of India

(d)  Reserve Bank of India

(e)  General Insurance Corporation of India

Answer: (c)

85. The Brabourne Stadium, which is India’s first permanent sporting venue, is situated in

(a)  Mumbai

(b)  New Delhi

(c)  Kolkata

(d)  Bengaluru

(e)  Chennai

Answer: (a)

86. A representative of the insurance company licensed by the State who solicits, negotiates or effects contact of insurance and provides services to the policyholder for the insurer, is known as

(a)  Insurance Trade

(b)  Insurance Agent

(c)  Insurance Broker

(d)  Stock Broker

(e)  Insurance Executive

Answer: (b)

87. Approximately of all anaemia cases worldwide are found to be caused due to

(a)  iron deficiency

(b)  potassium deficiency

(c)  calcium deficiency

(d)  iodine deficiency

(e)  sodium deficiency

Answer: (a)

88. The International Worker’s Day (also known as Labour Day) is observed every year on

(a)  April 1

(b)  May 1

(c)  May 31

(d)  May 10

(e)  April 10

Answer: (b)

89. ‘Lee Kuan Yew’ who recently died at the age of 91 years, was the founder and first Premier of

(a)  Japan

(b)  Hong Kong

(c)  South Korea

(d)  Singapore

(e)  Australia

Answer: (d)

90. The PMJDY, launched by the government of India, is a massive

(a)  tribal welfare programme

(b)  financial literacy programme

(c)  infrastructure development programme

(d)  financial inclusion programme

(e)  social security programme

Answer: (d)

91. The largest producer of cotton in the world, is

(a)  Malaysia

(b)  Mexico

(c)  India

(d)  Canada

(e)  China

Answer: (c)

92. Who amongst the following has recently been named as the head of the task force set-up by the government of India to define poverty and prepare a roadmap to alleviate it?

(a)  Dr. Bimal Jalan

(b)  Sindhushree Khullar

(c)  Arvind Panagariya

(d)  VK Saraswat

(e)  Bibek Debroy

Answer: (c)

93. ‘The Sun’ is a daily tabloid newspaper published in

(a)  the USA and Canada

(b)  the USA

(c)  Russia

(d)  Europe

(e)  the UK and Ireland

Answer: (e)

94. An application for obtaining an insurance cover or for obtaining quotations of the premium chargeable, is known as the

(a)  Proposal

(b)  Brochure

(c)  Catalogue

(d)  Pamphlet

(e)  Prospectus

Answer: (a)

95. ‘Dublin’ is the national capital of the

(a)  Republic of Israel

(b)  Republic of Iran

(c)  Republic of Iraq

(d)  Republic of Ireland

(e)  Republic of Iceland

Answer: (d)

96. ‘Playing It My Way’ is an autobiography of

(a)  Yuvraj Singh

(b)  Saurav Ganguly

(c)  Sunil Gavaskar

(d)  Sachin Tendulkar

(e)  Kapil Dev

Answer: (d)

97. Singapore’s Central Bank and Financial Regulatory Authority, is known as

(a)  Monetary Authority of Singapore

(b)  Reserve Bank of Singapore

(c)  Other than those given as options

(d)  State Bank of Singapore

(e)  Federal Reserve of Singapore

Answer: (a)

98. The Chutak hydroelectric plant is a run-of-the-river power project on the Suru river (a tributary of Indus) in

(a)  Kathua district (Jammu & Kashmir)

(b)  Leh district (Jammu & Kashmir)

(c)  Baramulla district (Jammu & Kashmir)

(d)  Doda district (Jammu & Kashmir)

(e)  Kargil district (Jammu & Kashmir)

Answer: (e)

99. A life insurance policy wherein the policyholder receives a fixed amount at specific intervals throughout the duration of the policy, is known as

(a)  Whole Life Plan

(b)  Unit Linked Insurance Plan

(c)  Money-back Policy

(d)  Fixed Term Plan

(e)  Double Cover Plan

Answer: (c)

100. The ‘Bandipur National Park’ established in 1974 as a tiger reserve under ‘Project Tiger’ is located in the Indian State of

(a)  Tamil Nadu

(b)  Kerala

(c)  Karnataka

(d)  Telangana

(e)  Andhra Pradesh

Answer: (c)

101. According to the recent data compiled by the World Steel Association (WSA), India is ranked as

(a)  third largest producer of steel in the world

(b)  fourth largest producer of steel in the world

(c)  fifth largest producer of steel in the world

(d)  largest producer of steel in the world

(e)  second largest producer of steel in the world

Answer: (b)

102. The Insurance Regulatory and Development Authority of India (IRDAI) is an autonomous apex

(a)  consultative body

(b)  statutory body

(c)  advisory body

(d)  cooperative body

(e)  corporate body

Answer: (b)

103. Who amongst the following is the first Indian women badminton player to achieve world number one ranking?

(a)  Meena Shah

(b)  Jwala Gutta

(c)  Saiina Nehwal

(d)  Sania Mirza

(e)  Aditi Mutatkar

Answer: (c)

104. At the 62nd National Film Awards, the award for the Best Feature Film has been given to the film

(a)  Chotushkone

(b)  Queen

(c)  Mary Kom

(d)  Court

(e)  Haider

Answer: (b)

105. The minimum lock-in period under the recently re-launched ‘Kisan Vikas Patra’ scheme, is

(a)  one year and six months

(b)  one year

(c)  two years and six months

(d)  two years

(e)  two years and three months

Answer: (c)

106. ‘Aviva India’ is an Indian life insurance company. It is a joint venture between Aviva plc. a British insurance company, and the Indian conglomerate?

(a)  Bajaj Group

(b)  Dabur Group

(c)  Kotak Mahindra Group

(d)  Tata Group

(e)  Bharati Group

Answer: (b)

107. ‘Steve Smith’ is associated with the game of

(a)  Cricket

(b)  hockey

(c)  football

(d)  tennis

(e)  wrestling

Answer: (a)

108. ‘Itanagar’ is the capital of the Indian State of

(a)  Assam

(b)  Meghalaya

(c)  Tripura

(d)  Arunachal Pradesh

(e)  Manipur

Answer: (d)

109. ‘Kedarnath Main’ and ‘Kedarnath Dome’ are two mountains in the Gangotri Group of Peaks in the Indian State of

(a)  Uttarakhand

(b)  Arunachal Pradesh

(c)  Uttar Pradesh

(d)  Meghalaya

(e)  Jammu & Kashmir

Answer: (a)

110. Which of the following organizations has recently been selected for the Gandhi Peace Prize 2014?

(a)  Vikram Sarabhai Space Centre

(b)  Indian Space Research Organization

(c)  Satish Dhawan Space Centre

(d)  Other than those given as options

(e)  Indian Institute of Space Science and Technology

Answer: (b)

111. The person, corporation or trust named in the insurance policy as the recipient of insurance proceeds upon the death of the life insured, is known as

(a)  Key man

(b)  Franchisee

(c)  Insured

(d)  Nominee

(e)  Beneficiary

Answer: (e)

112. Who amongst the following has recently been conferred the ‘Stockholm Water Prize 2015’ for his innovative water restoration efforts and extraordinary courage to empower communities in Indian villages?

(a)  Akhilesh Yadav

(b)  Sunder Lal Bahuguna

(c)  Naseeruddin Shah

(d)  Rajendra Singh

(e)  Chandi Prasad Bhatt

Answer: (d)

113. The ‘World Food Programme’ is the food assistance branch of the United Nations and is the world’s largest humanitarian organization addressing hunger and food security. It is headquartered in

(a)  Paris (France)

(b)  Vienna (Austria)

(c)  New York (USA)

(d)  Geneva (Switzerland)

(e)  Rome (Italy)

Answer: (e)

114. Which of the following institutes, presided over by the Union Minister of Agriculture, is an autonomous body responsible for coordinating agricultural education and research in India?

(a)  Indian Council of Agricultural Research

(b)  Indian Agricultural Research Institute

(c)  Indian Institute of Agriculture Research and Education

(d)  Institute of Agricultural Research in India

(e)  Indian Council of Agricultural Research and Education

Answer: (a)

115. With the government notifying changes in the Public Provident Scheme (PPF), investors can now deposit up to

(a)  Rs 1.5 lakh annually

(b)  Rs 1.00 lakh monthly

(c)  Rs 1.5 lakh quarterly

(d)  Rs 1.00 lakh quarterly

(e)  Rs 1.00 lakh annually

Answer: (a)

116. An account can be opened under the recently launched ‘Sukanya Samridhi Y Yojana’ at any time from the birth of a girl child till she attains the age of

(a)  fifteen years

(b)  eighteen years

(c)  twenty-one years

(d)  thirteen  years

(e)  ten years

Answer: (e)

117. An unstamped document issued in advance by an insurance company pending the issue of the policy, which is normally required if the policy cannot, for some reason or other, be issued straight away, is known as a

(a)  Commercial Invoice

(b)  Covering Letter

(c)  Draft Policy

(d)  Cover Note

(e)  Pro Term Policy

Answer: (d)

118. The official currency of Philippines, is

(a)  Philippine Ringgit

(b)  Philippine Shekel

(c)  Philippine Peso

(d)  Philippine Won

(e)  Philippine Riyal

Answer: (c)

119. The ‘NPS’ launched by the government of India with effect from January 1, 2004 is a defined-contribution

(a)  Mutual Fund System

(b)  Accident Insurance Scheme

(c)  Pension System

(d)  Life Insurance System

(e)  Fixed Deposit Scheme

Answer: (c)

120. Who amongst the following was the Brand Ambassador for the 2015 ICC World Cup tournament?

(a)  Shane Warne (Australia)

(b)  Brian Lara (West Indies)

(c)  Sachin Tendulkar (India)

(d)  Steve Waugh (Australia)

(e)  Betty Wilson (Australia)

Answer: (c)

Part IV Numerical Aptitude

121. In 2004, the total monthly salary of A and B together was Rs 18000. In 2005, monthly salary of A and B increased by 14% and 20% respectively from previous year. After the given increment, A’s salary became 76% of B’s salary. What was A’s salary in 2004 (i.e. before the mentioned increment of 2005)?

(a)  Rs 7500

(b)  Rs 8000

(c)  Rs 6500

(d)  Rs 8500

(e)  Rs 6000

Answer: (b)

Directions (Q. Nos. 122-126) Refer to the table and answer the given questions :

122. If the number of projects handled by company A increased by 25% from 2008 to 2009 and the number of projects handled by company E decreased by 35% from 2008 to 2009, then what was the total number of projects handled by companies A and E together in 2009?

(a)  377

(b)  381

(c)  393

(d)  363

(e)  369

Answer: (b)

123. Number of projects handled by company E increased by what percent from 2004 to 2006?

(a)  65%

(b)  55%

(c)  57.5%

(d)  52.5%

(e)  60%

Answer: (b)

124. All the given companies handled only two types of projects during all the given years-Governmental and Non-governmental. If the respective ratio between total number of Governmental projects and Non-governmental projects handled by all the given companies together in 2007 is 13 : 8, then total how many Governmental projects were handled by all the given companies together in 2007?

(a)  533

(b)  494

(c)  585

(d)  547

(e)  559

Answer: (d)

125. What is the difference between average number of projects handled by company B in 2004 and 2005 together and average number projects handled by company D in 2007 and 2008 together?

(a)  52

(b)  43

(c)  54

(d)  48

(e)  44

Answer: (b)

126. Out of the total number of projects handled by company C during all the given years together, 61% were Governmental projects and the remaining were Non-governmental projects. What is the total number of Non-governmental projects handled by company C during all the given y ears together?

(a)  321

(b)  373

(c)  351

(d)  339

(e)  343

Answer: (e)

Directions (Q. Nos. 127-131) What will come in place of question marks in the given numbers series?

127. 6   11    31      121    601    ?

(a)  3620

(b)  3000

(c)  3606

(d)  3601

(e)  3005

Answer: (d)

128. 8   16   64      384    ?        30720

(a)  2064

(b)  2160

(c)  2564

(d)  3102

(e)  3072

Answer: (e)

129. 101   103   109    121    141    ?

(a)  154

(b)  198

(c)  166

(d)  171

(e)  180

Answer: (d)

130. 2    14   38      86      ?        374

(a)  182

(b)  176

(c)  216

(d)  194

(e)  252

Answer: (a)

131. 10   28    63      132    252    ?

(a)  458

(b)  356

(c)  480

(d)  440

(e)  340

Answer: (d)

132. An interest of Rs 8384 is received when a certain sum is invested for 4 years in Scheme A which offers simple interest at 8% per annum. When the same sum of money is invested for 6 years in scheme B which also offers simple interest at a certain rate, the amount received is Rs 39562, what is the rate of interest offered by scheme B?

(a)  8.5%

(b)  7.25%

(c)  7.5%

(d)  8.25%

(e)  9.5%

Answer: (a)

133. A vessel was containing 80 L of pure milk. 16 L of pure milk was taken out and replaced with equal amount of water. 16 L of newly formed mixture of water and milk was taken out and then 24 L of water was added to the mixture. What is the respective ratio between the quantity of milk and water in the final mixture?

(a)  34 : 23

(b)  34 : 21

(c)  28 : 23

(d)  32 : 21

(e)  32 : 23

Answer: (e)

134. Pihu’s monthly income is Rs 39500. She spends 9/20 her monthly income on household expenditures and 24% of the monthly income on her children’s school fees. If 40% of the remaining monthly income she donates to NGO, what is the amount left with her in a month? (If there is no other expenditure)

(a)  Rs 7167

(b)  Rs 7594

(c)  Rs 7862

(d)  Rs 7347

(e)  Rs 7025

Answer: (d)

135. A group of workers could complete a piece of work in 84 days. If there were 6 more workers, it would have taken 12 days less to finish the same piece of work. What was the actual strength of workers?

(a)  42

(b)  36

(c)  48

(d)  32

(e)  40

Answer: (b)

136. A retailer bought 30 kg of rice at a discount of 20% on the market price. Besides, he was given 8 kg of rice free of cost, by the wholesaler for purchasing a bulk quantity. If the retailer sold the entire quantity of rice at the marked price to his customers, then what was his profit percent?

(a) 

(b)

(c) 

(d) 

(e) 

Answer: (c)

137. X is greater than Y by 12 and the respective ratio between X and Y is 3 : 2. What is the sum of third number Z and X, if Z is 5/15 of Y?

(a)  45

(b)  40.25

(c)  44

(d)  43.5

(e)  48.5

Answer: (c)

138. Out of Rs 8000, Gopal invested a certain sum in scheme A and the remaining sum in scheme B for two years. Both the schemes offer compound interest (compounded annually). The rates of interest of scheme A and B are 10% per annum and 20% per annum respectively. If the total amount accrued by him after two years from both the schemes together was rs 10600, then what was the amount invested in scheme B?

(a)  Rs 4000

(b)  Rs 4800

(c)  Rs 5200

(d)  Rs 3600

(e)  Rs 4400

Answer: (a)

139. The respective ratio of Avinash’s present age and Shashi’s present age is 9 : 5. Avinash is 54 years old at present. How many years ago the respective ratio of their ages was 7 : 3?

(a)  12 yr

(b)  6 yr

(c)  9 yr

(d)  15 yr

(e)  18 yr

Answer: (a)

140. A car travels from city A to city B at an average speed of 60 km/h and reaches city B on time. If the car reduces its speed to 50 km/h, it takes 16 min more to reach city B. What is the distance between city A and city B?

(a)  80 km

(b)  85 km

(c)  82 km

(d)  84 km

(e)  86 km

Answer: (a)

141. The diameter of a circle is equal to the diagonal of a square whose area is 784 m12. What is the area of the circle?

(a)  1186 m2

(b)  1232 m2

(c)  1272 m2

(d)  1324 m2

(e)  1264 m2

Answer: (b)

142. The price of 8 books and 24 registers is Rs 1760. If the price of one book is Rs 124 more than the price of one register, what is the total price of 4 books and 2 registers?

(a)  Rs 512

(b)  Rs 674

(c)  Rs 756

(d)  Rs 640

(e)  Rs 544

Answer: (d)

143. What should be added to each of the numbers 13, 17 and 22 so that the resulting numbers are in continued proportion?

(a)  2

(b)  5

(c)  3

(d)  7

(e)  9

Answer: (c)

Directions (Q. Nos. 144-158) What will come in place of questions marks in the given questions.

144. 

(a)  1.75

(b)  1.5

(c)  0.115

(d)  0.85

(e)  0.95

Answer: (e)

145. 292 – 435 + ? = 1652 ×5

(a)  27

(b)  23

(c)  19

(d)  29

(e)  17

Answer: (*)

146. 10(1024.8 + 24.6 – 4.2) =?2 + 48

(a)  105

(b)  120

(c)  110

(d)  130

(e)  102

Answer: (e)

147. 

(a)  153

(b)  182

(c)  221

(d)  187

(e)  204

Answer: (d)

148. (0.6 + 0.4 + 10) (0.8 – 0.4 + 10) = ?

(a)  143.8

(b)  114.8

(c)  114.4

(d)  114

(e)  134.4

Answer: (c)

149. 

(a)  2.99

(b)  4.9

(c)  2.79

(d)  2.3

(e)  3.12

Answer: (a)

150. (42.25 ÷5 + 16.2 ÷ 0.2) × ? = 1906

(a)  16

(b)  8

(c)  12

(d)  10

(e)  14

Answer: (c)

151. (0.8)5 × (0.8)1.5 ÷ (0.8)0.5 = (0.2)3.5 × 2?

(a)  8

(b)  6.5

(c)  7

(d)  7.5

(e)  8.5

Answer: (c)

152. 6242.52 – 242.2 – ? = 5974.12

(a)  36.2

(b)  24.2

(c)  26.2

(d)  14.2

(e)  16.2

Answer: (c)

153. 

(a)  6

(b)  8

(c)  3

(d)  4

(e)  7

Answer: (c)

154. 

(a)  5

(b)  9

(c)  6

(d)  7

(e)  8

Answer: (d)

155. 34 × 80 + ?2 = 7500 – 4251

(a)  27

(b)  17

(c)  21

(d)  33

(e)  23

Answer: (e)

156. 

(a)  7

(b)  11

(c)  9

(d)  13

(e)  15

Answer: (d)

157. 

(a)  80

(b)  42

(c)  32

(d)  60

(e)  62

Answer: (a)

158. 

(a)  4356

(b)  2916

(c)  3136

(d)  3844

(e)  4096

Answer: (a)

159. A and B started a business together and the respective ratio between the investments of A and B was 5 : 9. After four months from the start of business, C joined and the respective ratio between the investments of B and C was 3 : 7. If the annual profit earned by them was Rs 7084, then what is C’s share of profit?

(a)  Rs 3542

(b)  Rs 3742

(c)  Rs 3112

(d)  Rs 3600

(e)  Rs 3412

Answer: (a)

160. The length and breadth of a rectangular lawn are 36 m and 20 m respectively. It has two roads, each 3 m wide and one parallel to the length and the other parallel to the breadth. What is the cost of gravelling the two roads at the rate of Rs 3?

(a)  Rs 471

(b)  Rs 499

(c)  Rs 459

(d)  Rs 501

(e)  Rs 477

Answer: (e)

Part V Computer Knowledge

161. Which one of the following would not be considered as a form of secondary storage?

(a)  Floppy disk

(b)  Optical disk

(c)  RAM

(d)  Flash drive

(e)  Hard disk

Answer: (c)

162. The processor is an example of computer

(a)  software

(b)  output unit

(c)  storage

(d)  program

(e)  hardware

Answer: (e)

163. A hexadecimal number is a number to the base

(a)  2

(b)  20

(c)  16

(d)  4

(e)  8

Answer: (c)

164. Java, in computer programming, is a

(a)  compiler

(b)  hardware device driver

(c)  low-level language

(d)  high-level language

(e)  programming mid-level language

Answer: (d)

165. Which of the following is not a binary number?

(a)  01010

(b)  11111

(c)  31121

(d)  00000

(e)  110110

Answer: (c)

166. What does the acronym WAN stand for?

(a)  Widest Area Network

(b)  Wild Area Network

(c)  Wide Area Networking

(d)  Wide Area Network

(e)  Wider Area Network

Answer: (d)

167. FTP is an acronym for

(a)  File Transaction Protocol

(b)  File Truncation Protocol

(c)  File Translation Protocol

(d)  File Transfer Protocol

(e)  File Transmission Protocol

Answer: (d)

168. Connections to the internet using a phone line and a modem are called …………. connections.

(a)  Digital

(b)  Dial-up

(c)  Broadband

(d)  Dish

(e)  Both (b) and (c)

Answer: (b)

169. Documents converted to …………. can be published to the web.

(a)  a doc file

(b)  HTTP

(c)  machine language

(d)  HTML

(e)  None of these

Answer: (d)

170. What is the extension of Microsoft Word document?

(a)  txt

(b)  doc

(c)  ppt

(d)  xls

(e)  psd

Answer: (b)

171. Outlooks Express is a (n)

(a)  protocol

(b)  search Engine

(c)  browser

(d)  network

(e)  e-main client

Answer: (e)

172. Printers and screens in computer system are common form of

(a)  processing device

(b)  computation units

(c)  input units

(d)  storage units

(e)  output units

Answer: (e)

173. LSI in chip technology stands for

(a)  Large-Scale Internet

(b)  Low-Scale Integration

(c)  Low-Scale Internet

(d)  Local-Scale Integration

(e)  Large-Scale Integration

Answer: (e)

174. ‘Ctrl + N’ in MS Word is used to

(a)  save document

(b)  start paragraph

(c)  open new document

(d)  close document

(e)  open as saved document

Answer: (c)

175. Connections to other documents or to other locations within a website, is known as

(a)  Hyper connects

(b)  Plug-ins

(c)  Hyperlinks

(d)  Filters

(e)  Bots

Answer: (c)

176. Checking whether a program functions correctly and then correcting errors, it is known as

(a)  Error-proofing a program

(b)  Default error-checking

(c)  De-erroring

(d)  Debugging

(e)  Decoding

Answer: (d)

177. Single-word reference to viruses, worms etc, is

(a)  harmware

(b)  virus

(c)  phish

(d)  malware

(e)  killer war

Answer: (d)

178. Converting the computer language of 1s and 0s to characters, the can be understood, is known as

(a)  Selecting

(b)  Creating clip-art

(c)  Decoding

(d)  Generating

(e)  Highlighting

Answer: (c)

179. Storage that returns its data after the power is turned off, is referred to as

(a)  Sequential storage

(b)  Direct storage

(c)  Non-destructive storage

(d)  Non-volatile storage

(e)  Volatile storage

Answer: (d)

180. A ‘compiler’ in computing means

(a)  a programmer

(b)  a person who compiles source program

(c)  keypunch operator

(d)  a gaming software

(e)  a program which translates source program into object program

Answer: (e)

181. What is ‘Windows Vista’?

(a)  Processor

(b)  Output device

(c)  Storage device

(d)  Input device

(e)  Operating system

Answer: (e)

182. Which of the following is not an operating system used in computer?

(a)  Windows 95

(b)  Window 2000

(c)  MS-DOS

(d)  Windows 3.1

(e)  Windows 98

Answer: (d)

183. ‘DTP’ is a computer abbreviation usually means

(a)  Digital Transmission Protocol

(b)  Desk Top Publishing

(c)  Document Type Processing

(d)  Document Transfer Processing

(e)  Data Type Programming

Answer: (b)

184. Which of the following is equivalent roughly to 1 billion bytes?

(a)  One Kilobyte

(b)  One Terabyte

(c)  One Gigabyte

(d)  One Megabyte

(e)  One Megabyte

Answer: (c)

185. The ALU and Control Unit, jointly is known as

(a)  RAM

(b)  CPU

(c)  PC

(d)  EPROM

(e)  ROM

Answer: (b)

186. The ……… port resembles a standard phone jack.

(a)  fire wire

(b)  modem

(c)  peripheral

(d)  serial

(e)  None of these

Answer: (b)

187. The ALU performs ……….. operations.

(a)  arithmetic

(b)  ASCII

(c)  algorithm-based

(d)  logarithm-based

(e)  None of these

Answer: (a)

188. A group of related records in a database, is called a(n)

(a)  Object

(b)  Memo

(c)  Table

(d)  Record

(e)  None of these

Answer: (c)

189. What is the generation of computers which are built with microprocessors?

(a)  Third

(b)  Fourth

(c)  First

(d)  Second

(e)  Prior to first

Answer: (b)

190. The digital telecommunication term ISDN is an abbreviation for

(a)  Internet Services Data Network

(b)  International Services Digital Network

(c)  Integrated Services Digital Network

(d)  Interactive Standard Dynamic Networks

(e)  Integrated Standard Digital Networks

Answer: (c)

191. How to specify cell range from A10 A 25 in MS Excel?

(a)  (A10-A25)

(b)  (A10 to A25)

(c)  (A10.A25)

(d)  (A10-A25)

(e)  (A10 A25)

Answer: (e)

192. It you change Windows 98 operating system to Windows XP, then it is known as

(a)  Update

(b)  Patch

(c)  Pull down

(d)  Push up

(e)  Upgrade

Answer: (e)

193. ALU and Control Unit of most of the computers are combined and are embedded on a single

(a)  Monochip

(b)  Control unit

(c)  Microprocessor

(d)  ALU

(e)  Microfilm

Answer: (c)

194. QWERTY is used with reference to

(a)  printer

(b)  monitor

(c)  keyboard

(d)  mouse

(e)  joystick

Answer: (c)

195. Microsoft Word is an example of

(a)  a processing device

(b)  an operating system

(c)  application software

(d)  system software

(e)  an input device

Answer: (c)

196. File type in computing can be represented by

(a)  file identifier

(b)  file text

(c)  file name

(d)  file object

(e)  file extension

Answer: (e)

197. Which is a function key?

(a)  Esc

(b)  F12

(c)  Ins

(d)  Delete

(e)  Ctrl

Answer: (b)

198. A …………. computer is also referred to as a laptop computer.

(a)  mini

(b)  notebook

(c)  PDA

(d)  desktop

(e)  tablet

Answer: (b)

199. …………… is the unauthorized copying and distribution of software.

(a)  Cracking

(b)  Plagiarism

(c)  Software piracy

(d)  Software literacy

(e)  Hacking

Answer: (c)

200. Which of the following devices sends and receives data over telephone lines to and from computes?

(a)  Expansion slot

(b)  Modem

(c)  Speaker

(d)  Printer

(e)  Sound card

Answer: (b)

National Insurance Company Administrative Officers Examination Held on 4-4-2015 Reasoning Question Paper With Answer Key

National Insurance Company Administrative Officers Examination Held on 4-4-2015 Reasoning
National Insurance Company Administrative Officers Examination Held on 4-4-2015 Reasoning Question Paper With Answer Key

National Insurance Company Administrative Officers Examination Held on 4-4-2015

Reasoning

Directions-(Q. 1-5) Study the information carefully and answer the questions-

   S, T, U, V, W, X, Y and Z are sitting around a circular area at equal distance between each other, but not necessarily in the same order. .Some of the people are facing the centre while some face outside. (i.e., in a direction opposite tot the centre.)

   Note- Facing the same direction means if one faces the centre then the other also faces the centre and vice-versa. Facing opposite direction means if one person faces the centre then the other faces outside and vice-versa.

    U sits third to the right of X. X faces the centre. Only three  people sit between U and V. Z sits second to the right of V. W sits third to the left of S. S is an immediate neighbour of X. T sits second to the right of W. T is not an immediate neighbour of U. Immediate neighbours of T face opposite directions (i.e., if one neighbur faces the centre other faces outside and vice-versa.). U sits to the immediate left of Y. U and S face opposite directions T faces the same direction as W.

1. Who sits exactly between W and T, when counted from the right of W?

(A)  Y

(B)  Z

(C)  V

(D)  X

(E)  S

Answer: (B)

2. Four of the following five are alike in a certain way based on the given seating arrangement and so form a group. Which is the one that does not belong to that group?

(A)  Y

(B)  W

(C)  X

(D)  U

(E)  Z

Answer: (A)

3. What is X’s position with respect to Z?

(A)  Third to the right

(B)  Third to the left

(C)  Immediate right

(D)  Second to the left

(E)  Immediate left

Answer: (A)

4. Who sits second to the left of Y ?

(A)  T

(B)  W

(C)  S

(D)  Z

(E)  X

Answer: (B)

5. How many people in the given arrangement face outside ?

(A)  Three

(B)  One

(C)  Two

(D)  Five

(E)  Four

Answer: (A)

Directions- (Q. 6 and 7) Study the given information and answer the given questions-

   Following are few observations of a year-long study on mediation conducted on 300 people in the age group of 30-50 years-

(a) A specific form of mediation is a good way to increase creativity even in middle-aged people.

(b) Meditating regularly for even 15-20 minutes leads to 40-55% reduction in symptoms of stress and depression.

(c) Another form of meditation helps reduce cholesterol level, which further helps to maintain normal blood pressure.

6. Which of the following can be inferred from the given passage ?

(A)  Meditation is more effective than anti-depresents

(B)  There are than one types of meditation

(C)  Cholesterol level can b e controlled only through meditation

(D)  Meditation, to be fruitful, is to be done for a particular time only, i.e., meditating less than or more than the standard time is harmful

(E)  Mediation is not helpful for anything other than improving one’s creativity

Answer: (A)

7. Which of the following cannot be a finding of the given study on meditation ?

(A)  Regular meditation leads to a considerable increase in energy  levels of people, particularly of those in the age group of 35-45 years

(B)  Meditation improves the body metabolism, which is largely affected b y the cholesterol level in the body

(C)  106 at-risk adolescent in high schools reduced their levels of stress, anxiety and hyperactivity when practicing the Transcendental Meditation technique for two months at school

(D)  As a result of low stress level, insomnia was reduced by 42% in people who practiced meditation for three weeks

(E)  128 people in professions like designing, writing or journalism performed better when they practiced regular meditation

Answer: (E)

Directions- (Q. 8-17) In this question, relationship between different elements is shown in the statements. The statements are followed by conclusions. Study the conclusions based on the given statement and select the appropriate answer.

8. Statements :

L < M < N; M = O > P < Q; R > Q

Conclusions : I. L < P

II. P < N

(A)  Neither conclusion I nor II is true

(B)  Either conclusion I or II is true

(C)  Only conclusion II is true

(D)  Both conclusion I and II are true

(E)  Only conclusion I is true

Answer: (C)

9. Statements :

W < X < Y = Z < A; B < C = W

Conclusions : I. C < A

II. B > Z

(A)  Neither conclusion I nor II is true

(B)  Either conclusion I or II is true

(C)  Only conclusion II is true

(D)  Both conclusion I and II are true

(E)  Only conclusion I is true

Answer: (E)

10. Statements :

L < M < N; M  = O > P < Q; R > Q

Conclusions : I. Q < N

II. P < R

(A)  Neither conclusion I nor II is true

(B)  Either conclusion I or II is true

(C)  Only conclusion II is true

(D)  Both conclusion I and II are true

(E)  Only conclusion I is true

Answer: (C)

11. Statements :

D = E > F > G < H ; G < J < K

Conclusions : I. H < D

II. K > H

(A)  Neither conclusion I nor II is true

(B)  Either conclusion I or II is true

(C)  Only conclusion II is true

(D)  Both conclusion I and II are true

(E)  Only conclusion I is true

Answer: (A)

12. Statements :

S < T = U < V; X = W < S

Conclusions : I. X < V

II. X = V

(A)  Neither conclusion I nor II is true

(B)  Either conclusion I or II is true

(C)  Only conclusion II is true

(D)  Both conclusion I and II are true

(E)  Only conclusion I is true

Answer: (E)

13. Statements : Some tents are buildings. some buildings are chairs. Some chairs are windows.

Conclusions :

I. Some windows are buildings.

II. Some windows are tents.

(A)  Neither conclusion I nor II is true

(B)  Either conclusion I or II is true

(C)  Only conclusion II is true

(D)  Both conclusion I and II are true

(E)  Only conclusion I is true

Answer: (A)

14. Statements : All cups are benches. Some benches are drums. All drums are kites.

Conclusions:

I. Some kites are cups.

II. Some drums are cups.

(A)  Neither conclusion I nor II is true

(B)  Either conclusion I or II is true

(C)  Only conclusion II is true

(D)  Both conclusion I and II are true

(E)  Only conclusion I is true

Answer: (A)

15. Statements : All silver are gold. All aluminium are gold. Some silver aluminium.

Conclusions :

I. Some gold is both silver and aluminium.

II. All gold can be aluminium.

(A)  Neither conclusion I nor II is true

(B)  Either conclusion I or II is true

(C)  Only conclusion II is true

(D)  Both conclusion I and II are true

(E)  Only conclusion I is true

Answer: (E)

16. Statements : All rings are necklaces. All those necklaces which are not rings are earrings. No earring is a bracelet.

Conclusions :

I. Some bracelets are necklaces.

II. No ring is a bracelet.

(A)  Neither conclusion I nor II is true

(B)  Either conclusion I or II is true

(C)  Only conclusion II is true

(D)  Both conclusion I and II are true

(E)  Only conclusion I is true

Answer: (C)

17. Statements : All cheques are notes. All notes are tyres. All tyres are books. All books are rods.

Conclusions :

I. Some rods are notes

II. Some books are tyres.

(A)  Neither conclusion I nor II is true

(B)  Either conclusion I or II is true

(C)  Only conclusion II is true

(D)  Both conclusion I and II are true

(E)  Only conclusion I is true

Answer: (D)

18. In which direction is Mahatmaji’s statue facing?

I. The statue is towards the northern end of the city.

II. The statue’s shadow falls towards east at 5 O’clock in the evening.

(A)  The data in either in Statement I alone or statement II alone sufficient to answer the question

(B)  The data in statement I alone are sufficient to answer their question, while the data in statement II alone are not sufficient to answer the question

(C)  The data in both the statements I and II together are necessary to answer the question

(D)  The data in both the statements I and II together are not sufficient to answer the question

(E)  The data in statement II alone are sufficient to answer the question, while the data in statement I alone are not sufficient to answer the question

Answer: (D)

19. What is the total number of pupils in the final year class?

I. The number of boys in the final year class is twice as much as the number of girls in that class.

II. The sum of the ages of all the pupils in the class is 399 years and their average age is 19 years.

(A)  The data in either in Statement I alone or statement II alone sufficient to answer the question

(B)  The data in statement I alone are sufficient to answer their question, while the data in statement II alone are not sufficient to answer the question

(C)  The data in both the statements I and II together are necessary to answer the question

(D)  The data in both the statements I and II together are not sufficient to answer the question

(E)  The data in statement II alone are sufficient to answer the question, while the data in statement I alone are not sufficient to answer the question

Answer: (E)

20. How many floors are there in the building (including the ground floor) ?

I. The area in which the building stands has a rule that no building can have more than six floors (including the ground floor).

II. Three families reside in the building. No two families live immediately above or below each other.

(A)  The data in either in Statement I alone or statement II alone sufficient to answer the question

(B)  The data in statement I alone are sufficient to answer their question, while the data in statement II alone are not sufficient to answer the question

(C)  The data in both the statements I and II together are necessary to answer the question

(D)  The data in both the statements I and II together are not sufficient to answer the question

(E)  The data in statement II alone are sufficient to answer the question, while the data in statement I alone are not sufficient to answer the question

Answer: (E)

21. How many people read both Economic Times and Financial Express ?

I. Out of 500 readers, 200 read Financial Express, 220 read Economic Times and 50 read Indian Expresses.

II. Out of total 500 readers, 220 read Economic Times, 200 reads Financial Express and 50 read neither.

(A)  The data in either in Statement I alone or statement II alone sufficient to answer the question

(B)  The data in statement I alone are sufficient to answer their question, while the data in statement II alone are not sufficient to answer the question

(C)  The data in both the statements I and II together are necessary to answer the question

(D)  The data in both the statements I and II together are not sufficient to answer the question

(E)  The data in statement II alone are sufficient to answer the question, while the data in statement I alone are not sufficient to answer the question

Answer: (A)

22. In which year did Ranjana complete her graduation ?

I. According to Ranjana’s father she graduated after March 1989 but before March 1993.

II. The brother of Ranjana remembers correctly that she did her graduation after March 1991 but before March 1994 and the year of her graduation was an even number.

(A)  The data in either in Statement I alone or statement II alone sufficient to answer the question

(B)  The data in statement I alone are sufficient to answer their question, while the data in statement II alone are not sufficient to answer the question

(C)  The data in both the statements I and II together are necessary to answer the question

(D)  The data in both the statements I and II together are not sufficient to answer the question

(E)  The data in statement II alone are sufficient to answer the question, while the data in statement I alone are not sufficient to answer the question

Answer: (C)

Directions- (Q. 23-27) Study the following information carefully and answer the given questions-

   A word and number arrangement machine when given an input line of words and numbers rearranges them following a particular rule in each step. The following is an illustration of input and rearrangement. (All the numbers are two digit numbers).

Input : tent 13 wheat 21 ask 63 steal 49 hand 54 vast 85.

Step I : 85 wheat tent 13 21 ask 63 steal 49 hand 54 vast

Step II : 63 vast 85 wheat tent 13 21 ask steal 49 hands 54

Step III : 54 tent 63 vast 85 wheat 13 21 ask steal 49 hand

Step IV : 49 steal  54 tent 63 vast 85 wheat 13 21  ask hand

Step V : 21 hand 49 steal 54 tent 63 vast 85 wheat 13 ask

Step VI : 13 ask 21 hand 49 steal 54 tent 63 vast 85 wheat

Step VII : is the last step of the above input, as the desired arrangement is obtained.

    As per the rules followed in the given steps, find out in each of the following questions the appropriate step for the given input.

Input : Store 95 clean 56 tape 15 break 28 feet 35 wait 69 ice 71.

23. Which step number is the following output?

Output : 35 feet 56 Ice 69 store 71 tape 95 wait clean 15 break 28.

(A)  Step V

(B)  Step III

(C)  Step VI

(D)  Step IV

(E)  There is no such step

Answer: (A)

24. Which number/word be at sixth position from the right in step IV ?

(A)  15

(B)  wait

(C)  clean

(D)  95

(E)  28

Answer: (C)

25. How many elements (words/numbers) are there between ‘feet’ and ‘15’ as they appear in the second last step of the output?

(A)  Six

(B)  Seven

(C)  Five

(D)  Eight

(E)  Nine

Answer: (D)

26. Which of the following represents the position of ‘wait’ in the third step?

(A)  Ninth from the left

(B)  Sixth from the left

(C)  Eight from the right

(D)  Seventh from the right

(E)  Tenth from the right

Answer: (B)

27. Which number/ word would be at seventh position from the left in the step VI ?

(A)  ice

(B)  store

(C)  tape

(D)  71

(E)  69

Answer: (E)

Directions- (Q. 28-30) Study the following information carefully and answer the given questions-

    Point D is 14 m towards the West of point A. Point B is 4 m towards the South of point D. Point F is 9 m towards the South of point D. Point E is 7 m towards the East of point B. Point C is 4 m towards the North of point E. Point G is 4 m towards the South of point A.

28. If a person walks 5 m towards the North from point F and then takes a right turn, which of the following points would he reach first ?

(A)  G

(B)  D

(C)  F

(D)  A

(E)  C

Answer: (C)

29. Which of the following points are in a straight line ?

(A)  D, C, A

(B)  E, G, C

(C)  D, B, G

(D)  E, G, B

(E)  F, B, C

Answer: (A)

30. A is in which direction with respect to C?

(A)  East

(B)  West

(C)  North

(D)  South

(E)  Cannot be determined

Answer: (A)

Directions- (Q. 31-35) Study the following information and answer the questions-

   Eight friends- A, B, C, D, E, F, G and H live on eight different floors of a building but not necessarily in the same order. The lower most floor of the building is numbered one, the one above that is numbered two and so on till the topmost floor is numbered eight.

    A lives on floor numbered five. Only two people live between A and H. C lives  immediately above D. C lives on an odd numbered floor. Only one person lives between D and F. Only two people live between F and G. B lives on an odd numbered floor but not the lowermost floor. E lives on any of the floors above B.

31. Who amongst the following lives on floor numbered two ?

(A)  D

(B)  C

(C)  F

(D)  B

(E)  H

Answer: (E)

32. Four of the following five are alike as per the given arrangement and thus form a group. Which of the following does not belong to that group ?

(A)  BG

(B)  HF

(C)  AC

(D)  DA

(E)  ED

Answer: (D)

33. Who live exactly between the floors on which D and E live ?

(A)  C

(B)  A

(C)  B

(D)  No one

(E)  H

Answer: (A)

34. Who lives on the topmost floor (i.e., floor numbered eight) ?

(A)  D

(B)  F

(C)  Cannot be determined

(D)  C

(E)  E

Answer: (E)

35. Which of the following is true about G ?

(A)  G lives immediately below E.

(B)  Only two people live between G and B.

(C)  G lives on the lower most floor.

(D)  All the given statements are true.

(E)  Only one person live between G and A.

Answer: (C)

Directions (Q. 36-40) Following questions are based on the information given below-

(A) ‘P * Q means ‘P is father of Q’.

(B) ‘P – Q means ‘P is sister of Q’.

(C) ‘P + Q’ means ‘P is mother of Q’.

(D) ‘P/Q’ means ‘P is brother of Q’.

36. In the expression B + D * M/N. how is M related t B ?

(A)  Granddaughter

(B)  Son

(C)  Grandson

(D)  Daughter

(E)  None of these

Answer: (C)

37. Which of the following represents ‘J is son of F’?

(A)  J/R – T * F

(B)  J + R – T * F

(C)  J / M – N * F

(D)  Cannot be determined

(E)  None of these

Answer: (E)

38. Which of the following represents ‘R is niece of M’ ?

(A)  M/K * T – R

(B)  M – J + R – N

(C)  R – M * T/W

(D)  Cannot be determined

(E)  None of these

Answer: (B)

39. Among A, B, C, D and E, each having a different weight. D is not lighter than B and E is not heavier than A. C is not the heaviest. Who among them is the lightest ?

(A)  D

(B)  B

(C)  E

(D)  Data inadequate

(E)  None of these

Answer: (D)

40. In a certain code, DONE is written as ‘5139’ and ‘SEAL’ is written as ‘8942’. How is LOAD written in that code ?

(A)  2415

(B)  2145

(C)  2945

(D)  2182

(E)  None of these

Answer: (B)

National Insurance Company Ltd. Administrative Officers Examination Held on 12-4-2015 General Awareness Question Paper With Answer Key

National Insurance Company Ltd. Administrative Officers Examination Held on 12-4-2015 General Awareness
National Insurance Company Ltd. Administrative Officers Examination Held on 12-4-2015 General Awareness Question Paper With Answer Key

National Insurance Company Ltd. Administrative Officers Examination Held on 12-4-2015

General Awareness

1. Mohammad Nasheed who was arrested recently, is the former President of which country ?

(A)  Philippines

(B)  Myanmar

(C)  Bangladesh

(D)  Maldives

(E)  Malaysia

Answer: (D)

2. Which IT firm has been ranked as Top Employer in Europe ?

(A)  Abbott

(B)  Tata Consultancy Services

(C)  Bentley

(D)  HEINEKEN

(E)  Samsung

Answer: (B)

3. The country, which set up two commissions on February 10, to investigate human rights abuses and disappearances during nation’s decade long civil war-

(A)  Syria

(B)  Iraq

(C)  Nepal

(D)  Tunisia

(E)  Afghanistan

Answer: (C)

4. In ‘SIFI’, ‘S’ stands for-

(A)  Science

(B)  School

(C)  Systematically

(D)  Structural

(E)  System

Answer: (C)

5. Air India headquarters located at-

(A)  Hyderabad

(B)  Kolkata

(C)  Chennai

(D)  New Delhi

(E)  Ahmedabad

Answer: (D)

6. As per the World Bank’s Index of Ease of Doing Business 2014 India was ranked-

(A)  138th

(B)  189th

(C)  123rd

(D)  142nd

(E)  154th

Answer: (D)

7. How many Bank Accounts were opened under “Pradhan Mantri Jan Dhan Yojana” on its inaugural day ?

(A)  2.00 crores

(B)  1.25 crores

(C)  1.50 crores

(D)  1.00 crores

(E)  None of these

Answer: (C)

8. What is the operational range of Shaheen III?

(A)  2750 km

(B)  2500 km

(C)  2250 km

(D)  3000 km

(E)  3150 km

Answer: (A)

9. Ganga Talao is a crater lake situated in a secluded mountain area in the district of Savanne, deep in the heart of-

(A)  Indonesia

(B)  Maldives

(C)  Mauritius

(D)  Malaysia

(E)  Thailand

Answer: (C)

10. Which one of the following rivers of India is called ‘biological desert’ ?

(A)  Noyyal

(B)  Damodar

(C)  Bhiwani

(D)  Bhadar

(E)  Naramada

Answer: (B)

11. Researchers of which country have developed a candidate vaccine to protect against the deadly Ebola virus which will soon undergo human clinical trials in Africa ?

(A)  Sweden

(B)  China

(C)  Britain

(D)  Russia

(E)  India

Answer: (D)

12. Which of the following authorities is approached for insurance related complaints ?

(A)  Governing Body of Insurance Council (GBIC)

(B)  Insurance Regulatory and Development Authority of India (IRDA)

(C)  Insurance Ombudsman

(D)  National Insurance Corporation Limited (NICL)

(E)  United India Insurance Corporation (UIIC)

Answer: (B)

13. Which of the following states is the winner of the recently held Ranji Trophy 2014-15?

(A)  Tamil Nadu

(B)  Maharashtra

(C)  Mumbai

(D)  Karnataka

(E)  Vidarbha

Answer: (D)

14. The currency of Belgium is-

(A)  Belgian Franc

(B)  Euro

(C)  Dollar

(D)  Pound

(E)  Yen

Answer: (B)

15. Parliament has passed Insurance Laws (amendment) Bill, 2015 in Rajya Sabha on-

(A)  12 January, 2015

(B)  12 February 2015

(C)  10 February, 2015

(D)  10 March, 2015

(E)  12 March, 2015

Answer: (E)

16. On March 8, which of the following days is celebrated ?

(A)  Army Day

(B)  Women’s Day

(C)  Labour Day

(D)  AIDS Day

(E)  Navy Day

Answer: (B)

17. Kidambi Srikanth is associated with which of the following sports ?

(A)  Chess

(B)  Cricket

(C)  Badminton

(D)  Lawn Tennis

(E)  Hockey

Answer: (C)

18. According to the Insurance Amendment Bill 2015, a health insurance company is required to maintain an equity capital sum of how much ?

(A)  Rs 25 crores

(B)  Rs 22 crores

(C)  Rs 30 crores

(D)  Rs 25 crores

(E)  Rs 100 crores

Answer: (E)

19. According to the census 2011, what is the literacy rate in India ?

(A)  74.04%

(B)  65.46%

(C)  82.14%

(D)  61.80%

(E)  67.02%

Answer: (A)

20. National Insurance Academy (NIA) is situated at-

(A)  Hyderabad

(B)  New Delhi

(C)  Ahmedabad

(D)  Kochi

(E)  Pune

Answer: (E)

21. Which of the following countries has Copenhagen as its capital ?

(A)  Austria

(B)  Denmark

(C)  Argentina

(D)  Sweden

(E)  Germany

Answer: (B)

22. ‘The Substance and Shadow’ is an autobiography of which of the following eminent personality ?

(A)  R. K. Laxman

(B)  Dilip Kumar

(C)  Amitabh Bachchan

(D)  Lata Mangeshkar

(E)  Hema Malini

Answer: (B)

23. What is the maximum age limit to open Sukanya Samridhi Account ?

(A)  10 years

(B)  12 years

(C)  15 years

(D)  5 years

(E)  8 years

Answer: (A)

24. World Intellectual Property Organization (WIPO) head- quarters are located at-

(A)  New York, USA

(B)  Geneva, Switzerland

(C)  Paris, France

(D)  Vienna, Austria

(E)  Berlin, Germany

Answer: (B)

25. Match 10, 2015 is the foundation day of which organization ?

(A)  IIT Bombay

(B)  Life Insurance Corporation of India

(C)  CSIR

(D)  Public Health Foundation of India

(E)  Andhra Pradesh Foundation Day

Answer: (A)

26. The tennis legend who brought up his 1000th victory at tour level on January 11, 2015 in Brisbane, Australia, is-

(A)  David Ferrer

(B)  Roger Federer

(C)  Stanislas Wawrinka

(D)  Rafael Nadal

(E)  Novak Djokovic

Answer: (B)

27. Maximum shares in NICL is held by-

(A)  Scheduled Commercial Banks

(B)  Reserve Bank of India

(C)  Government of India

(D)  IRDA

(E)  Both (B) and (D)

Answer: (C)

28. The Union Budget 2015-16 announced additional surcharge for the super rich with income of over Rs 1 crore. At what rate will this surcharge be charged ?

(A)  1%

(B)  2%

(C)  3%

(D)  4%

(E)  6%

Answer: (B)

29. With reference to which sector of Indian economy did the Economic Survey say that it is suffering from “double financial representation” ?

(A)  Services sector

(B)  Infrastructure sector

(C)  Banking sector

(D)  Only (A), (B) and (C)

(E)  Insurance sector

Answer: (C)

30. Which of the following is the winner of the best picture for Oscar Awards 2015?

(A)  The Theory of Everything

(B)  The Grand Budapest Hotel

(C)  Boyhood

(D)  Birdman

(E)  Whiplash

Answer: (D)

31. As per the Income Tax Regulations, E-ring of the income tax returns is mandatory for individuals, including salaried tax payers, earning more than ………. taxable income during the financial

(A)  Rs 10.00 lakhs

(B)  Rs 7.50 lakhs

(C)  Rs 2 lakhs

(D)  Rs 5 lakhs

(E)  None of these

Answer: (D)

32. The full form of NBFC is-

(A)  Non-Banking Foreign Company

(B)  Non-Business Foreign Company

(C)  New-Business Financial Corporation

(D)  New-Business Finance Company

(E)  Non-Banking Financial Company

Answer: (E)

33. In which of the following countries, the next SAARC summit will be held ?

(A)  India

(B)  Pakistan

(C)  Malaysia

(D)  Sri Lanka

(E)  China

Answer: (B)

34. In insurance sector,, FDI is raised from 26% to-

(A)  47%

(B)  50%

(C)  49%

(D)  40%

(E)  60%

Answer: (C)

35. Which of the following firm has the highest revenue ?

(A)  Indian Oil Corporation (IOC)

(B)  Bharat Petroleum

(C)  Hindustan Petroleum

(D)  Reliance Industries

(E)  State Bank of India

Answer: (A)

36. Ranthambore National Park is situated in which of the following states ?

(A)  Gujarat

(B)  Uttarakhand

(C)  Karnataka

(D)  Assam

(E)  Rajasthan

Answer: (E)

37. Which amongst the following is not an insurance company functioning in India ?

(A)  ICICI prudential

(B)  ING Vysya

(C)  National Securities Depsitory Limited

(D)  New India Insurance Company

(E)  None of these

Answer: (C)

38. Which of the following is not related to the insurance sector ?

(A)  Indemnity

(B)  Coverage

(C)  Misuse Alert

(D)  Annuity

(E)  Life Insurance

Answer: (C)

39. Which of the following types of companies/ organizations issue ULIP ?

(A)  Insurance companies

(B)  Banks

(C)  NABARD

(D)  RBI

(E)  SIDBI

Answer: (A)

40. ‘Kepler 78b’ is the name given to a recently discovered-

(A)  Island in the Arctic ocean

(B)  Unidentifying flying object

(C)  Planet beyond Solar System

(D)  Comet in the Solar System

(E)  Particle tested as ‘Positive Boson’

Answer: (C)

National Insurance Company Limited Administrative Officer (AO) Preliminary Examination Held on June 5, 2017 Question Paper With Answer Key

National Insurance Company Limited Administrative Officer (AO) Preliminary Examination Held on June 5, 2017
National Insurance Company Limited Administrative Officer (AO) Preliminary Examination Held on June 5, 2017 Question Paper With Answer Key

National Insurance Company Limited Administrative Officer (AO) Preliminary Examination Held on June 5, 2017

Part I Quantitative Aptitude

 

Directions (Q. Nos. 1-5) In each of these questions a number series is given. In each series one number is missing. Find out the missing number.

1. 7, 10, 15, 24, 39, ?

(a)  52

(b)  50

(c)  63

(d)  62

(e)  None of these

Answer: (d)

2. 104, 300, 469, 613, 734, ?

(a)  982

(b)  715

(c)  834

(d)  755

(e)  None of these

Answer: (c)

3. 5, 9, 17, 33, 65, ?

(a)  127

(b)  97

(c)  129

(d)  87

(e)  None of these

Answer: (c)

4. 9, 21, 45, 81, 129, ?

(a)  187

(b)  199

(c)  177

(d)  189

(e)  None of these

Answer: (d)

5. 12, 16, 32, 68, 132, ?

(a)  196

(b)  232

(c)  276

(d)  213

(e)  None of these

Answer: (b)

Directions (Q. Nos. 6-10) Study the following table carefully and answer the given questions.

6. What is the average number of eggs produced by farm D over the years?

(a)  7703

(b)  7700

(c)  7704

(d)  7604

(e)  Other than those given as options

Answer: (e)

7. The eggs produced by farm B in the year 2003are approximately. What percent of the eggs produced by farm B over the years?

(a)  18%

(b)  16%

(c)  19%

(d)  20%

(e)  Other than those given as options

Answer: (a)

8. What is the respective ratio of the dozens of eggs produced by farm A to farm E in the year 2005?

(a)  59 : 61

(b)  43: 59

(c)  61 : 58

(d)  61 : 59

(e)  Other than those given as options

Answer: (d)

9. What is the respective ratio of the dozens of eggs produced by farms A, B and C together in the year 2000, to the dozens of eggs produced by farms D, E and F together in the same year?

(a)  60 : 40

(b)  61 : 49

(c)  49 : 61

(d)  61 : 59

(e)  7 : 61

Answer: (c)

10. What is the difference between the total number of eggs produced by all the farms together in the year 2001 and the year 2004?

(a)  8204

(b)  8208

(c)  8200

(d)  684

(e)  Other than those given as options

Answer: (d)

11. Tom started a business with Rs 52000 and after 4 months Harry joined him with Rs 39000. At the end of the year out of the total profits Harry received total Rs 20000 including one-fourth of the profits as commission for managing the business. What profit did Tom receive?

(a)  Rs 40000

(b)  Rs 20000

(c)  Rs 30000

(d)  Rs 50000

(e)  Other than those given as options

Answer: (a)

12. Three Science classes A, B and C take a Life Science test. The average score of class A is 83. The average score of class B is 76. The average score of class C is 85. The average score of class A and B is 79 and average score and average score of class Band C is 81. Then the average score of classes A, B and C is

(a)  80

(b)  80.5

(c)  81

(d)  81.5

(e)  82

Answer: (d)

13. Two squares are chosen at random ion a chess board, what is the probability that they have side in common?

(a)  1/32

(b)  1/18

(c)  1/63

(d)  1/36

(e)  1/56

Answer: (a)

Directions (Q. Nos. 14-18) Read the following graph and answer the questions  below.

14. What is the income of Company A?

(a)  Rs 21.6 crore

(b)  Rs 16.8 crore

(c)  Rs 22.4 crore

(d)  Rs 27.5 crore

(e)  Other than those given as options

Answer: (b)

15. What is the sum of net profit of company C, D and F?

(a)  Rs 12 crore

(b)  Rs 10.48 crore

(c)  Rs 12.46 crore

(d)  Rs 9.24 crore

(e)  Rs 8.64 crore

Answer: (d)

16. Find the difference between the total income and the total expenditure of all the six companies?

(a)  Rs 29.5 crore

(b)  Rs 25.8 crore

(c)  Rs 27.2 crore

(d)  Rs 20.6 crore

(e)  Other than those given as options

Answer: (d)

17. Profit of Company E is what percentage of the income of company C?

(a)  120%

(b)  88.14%

(c)  900%

(d)  40%

(e)  Other than those given as options

Answer: (b)

18. Income of Company B is how much percentage more than the expenditure of company C?

(a)  125%

(b)  150%

(c)  30%

(d)  14%

(e)  Other than those given as options

Answer: (e)

Directions (Q. Nos. 19-23) In thee questions two equations numbered I and II  have been given. You have to solve both the equations and choose the correct option.

Give answer

a. if x > y   b. if x ≥ y

c. if x < y   d. if x ≤ y

e. if x = y or relationship between x and y cannot be established

19. I. x2 + 3x – 4 = 0 II. 10y2 + 51y + 56 = 0

Answer: (e)

20. I. x2 + 18x + 65 = 0 II. 15y2 + 4y – 35 = 0

Answer: (c)

21. I. x2 + 10x + 25 = 0 II. y2 + 29y + 190 = 0

Answer: (a)

22. I. x2 – 6x – 40 = 0 II. y2 + 18y – 40 = 0

Answer: (e)

23. I. x2 + 5x – 66 = 0 II. 25y2 + 65y + 42 = 0

Answer: (e)

Directions (Q. Nos. 24-28) Study the following graph carefully and answer the questions.

24. If the water level of River-A in July is decreased by 20%j, then what will be the water level of River-A in July?

(a)  156 m

(b)  162 m

(c)  164 m

(d)  152 m

(e)  Other than those given as options

Answer: (c)

25. In which river and in which month respectively the water level is the highest?

(a)  River-C in August

(b)  River-D in September

(c)  River-A in July

(d)  River-B in August

(e)  Other than those given as options

Answer: (a)

26. What is the average water level of River-A in all the four months together?

(a)  224.50 m

(b)  212.25 m

(c)  210.75 m

(d)  222.25 m

(e)  other than those given as options

Answer: (c)

27. If the danger level of all the four rivers is above 215 m, which rive has not crossed the danger level in August but has crossed the danger level in July?

(a)  River-A

(b)  River-B

(c)  River-C

(d)  River-D

(e)  None of these

Answer: (b)

28. What was the respective ratio between the level of River-C in September and the water level of River-B in June?

(a)  91 : 101

(b)  94 : 101

(c)  51 : 103

(d)  31 : 101

(e)  92 : 101

Answer: (e)

29. The time taken by a boat to travel 117 km downstream is 9 hours and the same distance upstream is 13 hours. The speed of the stream is 1/4 of the sped of the boat. Find the distance travelled by the boat going upstream in 2 hours?

(a)  32 km

(b)  14 km

(c)  18 km

(d)  20 km

(e)  Other than those given as options

Answer: (c)

30. The ratio o f Smita’s age to her mother is 3 : 7 respectively and the different in their ages is 32 years. What will be the ratio of their ages 4 years hence?

(a)  4 : 19

(b)  5 : 14

(c)  3 : 20

(d)  7 : 15

(e)  3 : 8

Answer: (d)

31. The ratio of number of male and female employees in an office is 5 : 3. In the office there are Engineers and Managers. If 30% of the male employees are managers and 40% of the female employees are engineers then what is the ratio of number of Engineers in the company to the total number of employees?

(a)  47 : 43

(b)  33 : 50

(c)  47 : 80

(d)  52 : 75

(e)  Other than those given as options

Answer: (e)

32. In a mixture of 150 L, the ratio of milk to water is 2 : 1. What amount of water should be further added to the mixture so as the make the ratio of the milk to water 1 : 2 respectively?

(a)  145 L

(b)  160 L

(c)  180 L

(d)  150 L

(e)  Other than those given as options

Answer: (d)

33. The average speed of a bus is three-fifths of the average speed of a car which covers 3250 km in 65 hours. What is the average speed of the bus?

(a)  30 km/h

(b)  20 km/h

(c)  35 km/h

(d)  36 km/h

(e)  Other than those given as options

Answer: (a)

34. Mehul sold in item for Rs 5625 incurred a loss of 25%. At what price should he have sold the item to gain a profit of 25%?

(a)  Rs 9375

(b)  Rs 9357

(c)  Rs 9753

(d)  Rs 9573

(e)  Other than those given as options

Answer: (a)

35. A person spends 1/3 of his income on food. He spends the rest of the income in the ratio 4 : 1 on buying clothe and books respectively. He spent Rs 2400 on the books. Find the total income of man.

(a)  Rs 18000

(b)  Rs 24000

(c)  Rs 12000

(d)  Rs 14000

(e)  Other than those given as options

Answer: (a)

Reasoning Ability

36. A is wife of B. D is brother of A. P and Q are the children of E, who is wife of D. How is B related to Q?

(a)  Father

(b)  Grandmother

(c)  Uncle

(d)  Father-in-law

(e)  Cousin

Answer: (c)

Directions (Q. Nos. 37-39) Study the following information and answer the questions.

Lalit goes to his office from his house with his son by a bike. He drives the bike from point A. He drives 6 km towards South and reaches point B. Then he turns to his right and drives 8 km and reaches a bakery shop.

37. How far is bakeries shop from point A and in which direction?

(a)  10 km, South-West

(b)  11 km, North-East

(c)  12 km, South-West

(d)  13 km, South-East

(e)  Other than those given as options

Answer: (a)

38. From bakery store Lalit turns to his right and drives 4 km and reaches Point C. He then takes right turn and drives 5 km and reaches school and drops his son. What is the distance between bakery shop and school?

(a)  3.10 km

(b)  5.13 km

(c)  6.4 km

(d)  4.56 km

(e)  7.5 km

Answer: (c)

39. From the school he drives 5 km towards left then turns right and drives 2 km and reaches point D. Point D is in which direction of Point A?

(a)  North-West

(b)  South-West

(c)  South-East

(d)  North-East

(e)  North

Answer: (d)

Directions (Q. Nos. 40-44) Study the following information and answer t he given questions.

Maya, Pavan, Amit, Arvind, Raghu, Rajeev and Ashok live in different floors of the same building. The lowermost floor of the building is numbered 1, the floor immediately above that is numbered 2 and so on till the topmost floor is numbered 7.

Arvind lives in an even numbered floor. Maya does not live above Arvind’s floor. Pavan lives three floors above Rajeev’s floor. Only one person is living between the floor in which Rajeev and Maya are living. Ashok lives in an odd numbered floor. Rajeev does not live below Arvind. Amit lives in the floor immediately above Ashok’s floor.

40. Who lives in floor number 3?

(a)  Maya

(b)  Raghu

(c)  Rajeev

(d)  Amit

(e)  Ashok

Answer: (c)

41. Who is living immediately above Raghu’s floor?

(a)  Maya

(b)  Ashok

(c)  Amit

(d)  Pavan

(e)  Rajeev

Answer: (b)

42. Who lives on the topmost floor?

(a)  Amit

(b)  Ashok

(c)  Pavan

(d)  Raghu

(e)  Maya

Answer: (c)

43. On which floor is Amit living?

(a)  7

(b)  6

(c)  5

(d)  4

(e)  3

Answer: (b)

44. If all the people are arranged in the same order as they would appear in the dictionary, starting from top floor to bottom floor, position of how many person(s) will remain unchanged after the rearrangement?

(a)  None

(b)  One

(c)  Two

(d)  Three

(e)  More than three

Answer: (b)

Directions (Q. Nos. 45-49) Study the following information and answer the given questions.

Eight persons U, V, J, A, X, W, B and M are sitting around a square table in such a way that four of them sit at four corners of the square table while four sit in the m idle of each of the four sides. The ones who sit at the four corners face the centre while those who sit in the middle of the sides face outside.

A is sitting second to the right of U. U sits at one of the middle side of table. X sits second to the right of B. B is not the neighbour of U and A. There are three persons sits between A and M. J is not the neighbour of U. V sits second to the right of W.

45. Which of the following is not true with respect to the given seating arrangement?

(a)  Two persons sit between A and U.

(b)  B and M do not face each other in the seating arrangement.

(c)  X and A are immediate neighbours of each other.

(d)  W sits opposite to B.

(e)  U sits in middle of one of the sides of the square table.

Answer: (a)

46. Who amongst the following is sitting third to the right of V?

(a)  J

(b)  M

(c)  X

(d)  W

(e)  Cannot  be determined

Answer: (a)

47. How many people sit between X and W when counted in anti-clockwise direction from W?

(a)  None

(b)  One

(c)  Two

(d)  Three

(e)  More than three

Answer: (e)

48. Who amongst the following sits opposite to U?

(a)  A

(b)  W

(c)  V

(d)  J

(e)  Other than those given as options

Answer: (b)

49. What is the position of X with respect to J?

(a)  Immediate left

(b)  Second to the left

(c)  Third to the right

(d)  Immediate right

(e)  Second to the right

Answer: (a)

Directions (Q. Nos. 50-54) Study the given information carefully and answer the questions.

‘task night cheek black stage’ is coded as ’39 35 36 97 70’

‘two kind study cap lie’ is coded as ’92 56 25 84 83’

‘two lab night cheek cap’ is coded as ’35 16 56 39 84’

‘actor work task night’ is coded as ’35 77 36 42’

‘cap stage study king glass’ is coded as ’83 56 47 70 92’

(All the codes are two-digit codes only)

50. Which word is coded as ‘47’?

(a)  task

(b)  night

(c)  glass

(d)  black

(e)  stage

Answer: (c)

51. How can ‘cap black two’ be coded as?

(a)  84 56 97

(b)  56 39 83

(c)  77 25 39

(d)  42 39 35

(e)  Other than those given as options

Answer: (a)

52. Which of the following can possibly be coded as ’84 25 92’?

(a)  lie black stage

(b)  study two lie

(c)  actor night king

(d)  study lab work

(e)  two actor kind

Answer: (b)

53. For how many distinct words can their respective codes be determined?

(a)  8

(b)  9

(c)  10

(d)  6

(e)  7

Answer: (e)

54. What is the ratio of sum of the codes of ‘lie’ and ‘stage’ to the sum of the codes of ‘two’ and ‘cap’?

(a)  18 : 29

(b)  19 : 28

(c)  19 : 24

(d)  3 : 5

(e)  28 : 19

Answer: (b)

55. If M > N ≤ O < P and T < O > C ≥ Z are true, then which of the following options is definitely false?

(a)  M > O

(b)  C = T

(c)  O = Z

(d)  T < P

(e)  N ≥ S

Answer: (b)

56. If X > Y ≥ Z and A ≥ B > Y < M ≤ T are true, which of the following options is definitely false?

(a)  Z < X

(b)  T < X

(c)  A < Z

(d)  B > X

(e)  Z < T

Answer: (d)

Directions (Q. Nos. 57-59) Study the following information and answer the questions.

There are seven members, 4 men and 3 women in a family. They are A, B, C, D, E, F and G. B is an Athlete and father E. F is the paternal grandfather of E and is a Boxer. C, who is a wrestler, is daughter-in-law of D. G is E’s uncle and is an advocate. There is one Architect, one Journalist and one Pilot in the family C is not E’s mother. D is not married to B. There are three married couples in the family.

57. If F is the grandfather of a pilot, what is his daughter-in-law’ profession?

(a)  Journalist

(b)  Boxer

(c)  Advocate

(d)  Cannot be determined

(e)  Wrestler

Answer: (d)

58. If D is the Pilot and the Journalist is not a female then what E’s profession?

(a)  Architect

(b)  Journalist

(c)  Boxer

(d)  Wrestler

(e)  Athlete

Answer: (a)

59. If the Pilot does not have any child, then who is the Pilot?

(a)  A

(b)  C

(c)  E

(d)  Either A or B

(e)  Either B or C

Answer: (c)

Directions (Q. Nos. 60-64) Study the following information and answer the given questions.

Seven students P, Q, R, S, T U and W starts taking class from Monday to Sunday of the same week but not necessary in the same order.

T is taking class one of the day before Wednesday. There are three persons taking class between T and P.  neither Q nor R taking class on Wednesday. As many students taking class between P  and S is same as the students taking between S and Q. R has taken class immediately before the day on which U is taking the class.

60. How many students are taking class after W?

(a)  1

(b)  3

(c)  2

(d)  Cannot be determined

(e)  Other than those given as options

Answer: (b)

61. How many students were taking class between T and W?

(a)  None

(b)  1

(c)  2

(d)  3

(e)  More than three

Answer: (c)

62. S was taking class immediately after which day?

(a)  Monday

(b)  Tuesday

(c)  Wednesday

(d)  Friday

(e)  Saturday

Answer: (d)

63. Who among the following is taking class on Wednesday?

(a)  P

(b)  T

(c)  W

(d)  U

(e)  S

Answer: (d)

64. Who is taking class exactly between U and Q?

(a)  P

(b)  S

(c)  T

(d)  R

(e)  None

Answer: (a)

65. In a certain coding system, MICRO is written as NHDQP and WIDES is written as XHEDT, what should be the code for STAINS?

(a)  TUBJMR

(b)  RUBJMT

(c)  TSBHOR

(d)  TIJHASD

(e)  Other than those given as options

Answer: (c)

Directions (Q. Nos. 66-70) Study the following information and answer the questions.

Eight friends F, G, H, L, M, N and O and Q are sitting in a straight line facing North, but not necessary in the same order.

• H is sitting second to the right of N and L is sitting third to the right of H.

• M is sitting to the immediate right of F.

• Number of persons can sit between G and L is same as the numbers of persons are sitting between Q and F.

• O is sitting to the immediate right of L who is not an immediate neighbour of G.

• F is not an immediate neighbour of N.

66. If all the arranged persons are rearranged in ascending order from left to right, positions of how many of them will remain unchanged?

(a)  None

(b)  One

(c)  Two

(d)  Three

(e)  More than three

Answer: (a)

67. Which of the following pairs are sitting at the extreme ends?

(a)  N, M

(b)  Q, L

(c)  O, Q

(d)  Other than those given as options

(e)  Cannot be determined

Answer: (c)

68. Based on the given arrangement, Q is related to N and H is related to F in same way, as M is related to?

(a)  M

(b)  F

(c)  L

(d)  Q

(e)  Other than those given as options

Answer: (c)

69. How many persons are sitting between Q and M?

(a)  One

(b)  Two

(c)  Three

(d)  Four

(e)  More than four

Answer: (d)

70. Who is sitting third to the left of H?

(a)  Q

(b)  N

(c)  L

(d)  No one

(e)  Other than those given as options

Answer: (a)

Part III English

Directions (Q. Nos. 71-80) Read the following passage carefully and answer the question given below it.

Certain words here been printed in bold to help you locate them while answering some of the questions.

Once upon a time there was a boy whose name was Jack, and he lived with his mother on a common. They were very poor, and the o ld woman got her living by spinning, but jack was so lazy that he would do nothing but bask in the Sun in the hot weather, and sit  by the corner of the hearth in the winter-time.

So they called him Lazy Jack. His mother could not get him to do anything for her, and at last told him, one Monday, that if he did not begin to work for his  porridge, she would turn him out to get his  living as  he could.

This roused Jack, and he went out and hired himself for the next day to a neighbouring farmer for a penny; but as he was coming home, never having had any money before, he lost it in passing over a brook. “You stupid boy”, said his mother, “you should have put it in your pocket”. ‘I’ll do another time”, replied Jack. Well, the next day, Jack went out again and hired himself to a cow-kepper, who gave him a jar of milk for his day’s work. Jack took the jar and put it into the large pocket of his jacket, spilling it all, long before he got  home. “Dear me!” said the old woman; “you should have carried it one your head”. “I’ll do so another time,” said Jack. So the following day, Jack hired himself again to a farmer, who agreed to give him a cream cheese for his services.

In the evening Jack took the cheese, and went home with it one his head. By the time he got home the cheese was all spoilt, part of it being lost, and part matted with his fair. “You stupid lout”, said his mother, “you should have carried it very carefully in your hands”, “I’ll do so another time”, replied Jack. Now the next day, Lazy Jack again went out, and hired himself to a baker, who would give him nothing for his work but large tom-cat. Jack took the cat, and began carrying it very carefully in his hands, but in a short time pussy scratched him so much that he was compelled to let it go. When he got home, his mother said to him, “You silly fellow, you should have tied it with a string, and dragged it along after you”. “I’ll do so another time”, said Jack. So on the following day, Jack hired himself to a butcher, who rewarded him by the handsome present of a shoulder of mutton. Jack took the mutton, tied it with a string, and trailed it along after him in the dirt, so that by the time  he  head got home, the meat was completely spoilt. His mother was this time quite out of patience with him, for the next day was Sunday, and she was obliged to do with cabbage for her dinner. “You ninny-hammer”, said she to her son, “you should have carried it on your shoulder”. “I’ll do so another time”, replied Jack. Well, on the Monday Lazy Jack went once more and hired himself to a cattle-keeper, who gave him a donkey for his trouble. Now though Jack was strong he found it hard to hoist the donkey on this shoulders, but at last he did it, and began walking home slowly with his prize.

Now it so happened that in the course of his journey he passed a house where a rich man lived with his only daughter, a beautiful girl, who was deaf and dumb. And she had never laughed in her life, and the doctors said she would never speak till somebody made her laugh. So the father had given out that any man who made her laugh would receive her hand in marriage. Now this young lady happened to be looking out of the window when Jack was passing by with the donkey on his shoulders; and the poor beast with its legs sticking up in the air was kicking violently and heehawing with all its might. Well, the sight was so comical that she burst out into a great fit on laughter and immediately recovered her speech and hearing. Her father was overjoyed, and fulfilled his promise by marrying her to Lazy Jack, who was thus made rich gentlemen.

71. Why Jack was repeatedly asked to go for work by his mother?

(a)  Because Jack was too lazy to do any work at home.

(b)  Because the mother wanted her son to learn new work every day.

(c)  Because she was too old to earn living for her home.

(d)  Because she could not get any work for her son.

(e)  Because they were poor and she wanted her lazy son to contribute to their living.

Answer: (e)

72. What did Jack earn after hiring himself to a baker?

(a)  He was rewarded with a piece of mutton.

(b)  He was presented with a large tom-cat.

(c)  He was rewarded a jar of milk for his day’s work.

(d)  He was rewarded with a cream cheese.

(e)  He was rewarded with a coin.

Answer: (b)

73. Why according to the passage, his mother had to cook cabbage for the dinner?

(a)  Because there was nothing else to cook.

(b)  Because she wanted to teach her son a lesson for his continuous stupidity.

(c)  Because she was very annoyed by her son’s behaviour.

(d)  Because the meat was completely marred by her son.

(e)  Other than those given as options

Answer: (d)

74. Why the rich m an married his daughter to lazy Jack?

(a)  Because he promised to marry his daughter to the man who would make her laugh.

(b)  Because Jack was very sincere and honest.

(c)  Because Jack was comical enough to impress her daughter.

(d)  Because he got sympathetic towards Jack’s economic condition.

(e)  Because his daughter insisted him to do so.

Answer: (a)

75. Why did Jack carry the donkey on his shoulders?

(a)  He wanted to impress the rich man’s daughter.

(b)  His mother asked him to do so.

(c)  He was foolish as he did not known how to carry a donkey.

(d)  He followed what his mother advised him after last stupid act.

(e)  The donkey was rigid and reluctant to go with him.

Answer: (d)

76. What was the ultimate fate of lazy Jack?

(a)  He finally came out of his stupidity.

(b)  He was no more a lazy boy as he used to be earlier.

(c)  He got married to a rich girl and became a rich gentleman.

(d)  He was highly praised by his mother and the rich man.

(e)  Other than those given as options.

Answer: (c)

77. Choose the word which is most similar in meaning to the word ‘Hearth’ as used in the passage.

(a)  Roof

(b)  Haven

(c)  Warmth

(d)  Condo

(e)  Fireplace

Answer: (e)

78. Choose the word which is most similar in meaning to the word ‘Heehaw’ as used in the passage.

(a)  Cackle

(b)  Irritation

(c)  Malady

(d)  Torment

(e)  Lament

Answer: (a)

79. Choose the word which is most opposite in meaning to the word ‘Rouse’ as used in the passage.

(a)  Mobilize

(b)  Disturb

(c)  Concuss

(d)  Lull

(e)  Agitate

Answer: (d)

80. Choose the word which is most opposite in meaning to the word ‘Matted’ as used in the passage?

(a)  Kinky

(b)  Uncombed

(c)  Unsettle

(d)  Snarl

(e)  Untangled

Answer: (e)

Directions (Q. Nos. 81-90) Read each sentence to find out whether there is any grammatical or idiomatic error in it. The error, if any, will be in one part o the sentence. The number of that part is the answer. If there is no error, the answer is (e). (Ignore errors of punctuation, if any)

81. We got / everything ready for all / of them long / before they arrived.

(a)  We got

(b)  everything ready for all

(c)  of them long

(d)  before they arrived

(e)  No error

Answer: (a)

82. The thief was caught / after he has / disposed of / the stolen goods.

(a)  The thief was caught

(b)  after he has

(c)  disposed of

(d)  the stolen goods

(e)  No error

Answer: (b)

83. Delhi’s water supply/ system is certainly / as good as is expected / in a capital own.

(a)  Delhi’s water supply

(b)  system is certainly

(c)  as good as is expected

(d)  in a capital own

(e)  No error

Answer: (a)

84. He says that / a two-miles walk / always keeps him / healthy and fresh.

(a)  He says that

(b)  a two-miles walk

(c)  always keeps him

(d)  healthy and fresh

(e)  No error

Answer: (b)

85. Simmi took me to a hotel / and ordered for / two cups of coffee / which the waiter brought in no time.

(a)  Simmi took me to a hotel

(b)  and ordered for

(c)  two cups of coffee

(d)  which the waiter brought in no time

(e)  No error

Answer: (b)

86. The first European / sailor to come to India / in modern times / was Vasco-di-Gama.

(a)  The first European

(b)  sailor to come to India

(c)  in modern times

(d)  was Vasco-di-Gama

(e)  No error

Answer: (b)

87. When the was / under house arrest / he was debarred to send / a letter even to his wife.

(a)  When the was

(b)  under house arrest

(c)  he was debarred to send

(d)  a letter even to his wife

(e)  No error

Answer: (c)

88. In spite of his insincerity / he was awarded / cent-percent marks / in Mathematics.

(a)  In spite of his insincerity

(b)  he was awarded

(c)  cent-percent marks

(d)  in Mathematics

(e)  No error

Answer: (b)

89. Much to his fortune, he / married a girl who was / more tall and lovelier than / her elder sister.

(a)  Much to his fortune, he

(b)  married a girl who was

(c)  more tall and lovelier than

(d)  her elder sister

(e)  No error

Answer: (c)

90. She prefers intellectual to manual occupations, and / is not so fond for fancy work as many of the blind / children are; yet she/ is eager to join them in whatever they are doing.

(a)  She prefers intellectual to manual occupations, and

(b)  is not so fond for fancy work as many of the blind

(c)  children are; yet she

(d)  is eager to join them in whatever they are doing

(e)  No error

Answer: (b)

Directions (Q. Nos. 91-100) In the following passage there are blanks, each of which has been numbered. These numbers are printed below the passage and against each, five words are suggested, one of which fill the blanks appropriately. Find out the appropriate words without changing to meaning of the blanks.

The art of wood carving is very old craft in India. Since time immemorial, man had started (91) his home by using wood carving methods. They used to make dolls and toys for their children. Wood craft in India is a common heritage and is (92) to exist across all parts of the country. The traditional craftsmen used wood, stone, earth and colour for painting as raw materials, They (93) carved doors, door frames and brackets in palaces and temples. Images of Gods, Idols and Deities were also made of wood. Sacred woods were used in such creations. The wood carvers had to retain strict (94) throughout the period they were occupied in the work. There was no restriction on the usage of colour and varnish. We can see carved wooden symbols in the rural areas which reminds us of a tribal (95). Wood craft is still being used for carving toys and dolls. The modern wood carving has touched a new height of sculpturesque perfection. At maintain traditional local (96) at the same time.

The religious product includes images of Gods and legendary figures like Durga, Ganesha, etc. Some craftsmen carve a single piece of wood to form an object. These attempts are made for artistic (97). Some distinct forms and shapes of items of wood carft have come down from remote past and some (98) forms have been evolved maintaining the custom and simplicity.

In the past, these wood craft items, such as toys, dolls, idols, etc., were meant for local market. There is a great demand for these wooden products as decorative items. The mostly carry decorative value in modern society.

Some wood artisans make wooden masks (99) customary religious sentiments. The tools used for wood craft are mainly the carpenter’s tools, carving chisels and the wood. The woods that they use are soft and are easily available. There are thousands of wood craftsmen making pieces of wood carved items of (100) beauty.

91.

(a)  renovating

(b)  decorating

(c)  beautifying

(d)  colouring

(e)  farming

Answer: (b)

92.

(a)  established

(b)  found

(c)  inserted

(d)  set-up

(e)  tend

Answer: (b)

93.

(a)  beautifully

(b)  remarkably

(c)  splendidly

(d)  skilfully

(e)  effectively

Answer: (d)

94.

(a)  difficulty

(b)  tenacity

(c)  accuracy

(d)  austerity

(e)  frugality

Answer: (c)

95.

(a)  pedigree

(b)  culture

(c)  legacy

(d)  custom

(e)  hertiage

Answer: (a)

96.

(a)  zest

(b)  flavour

(c)  squirt

(d)  fragrance

(e)  spirit

Answer: (e)

97.

(a)  presentation

(b)  affirmation

(c)  illustration

(d)  demonstration

(e)  manifestation

Answer: (d)

98.

(a)  experimental

(b)  innovative

(c)  creative

(d)  imaginative

(e)  traditional

Answer: (e)

99.

(a)  exhibiting

(b)  illustrating

(c)  indicating

(d)  depicting

(e)  presenting

Answer: (a)

100.

(a)  wonderful

(b)  decorative

(c)  exceptional

(d)  anomalous

(e)  exemplary

Answer: (c)

National Insurance Company Ltd. Recruitment of Administrative Officers Examination-2015 Quantitative Aptitude Question Paper With Answer Key

National Insurance Company Ltd. Recruitment of Administrative Officers Examination-2015 Quantitative Aptitude
National Insurance Company Ltd. Recruitment of Administrative Officers Examination-2015 Quantitative Aptitude Question Paper With Answer Key

National Insurance Company Ltd. Recruitment of Administrative Officers Examination-2015

Quantitative Aptitude

 

1. A rectangular plot has a concrete path running in the middle of the plot parallel to the length of the plot. The rest of the plot is used as lawn, which has an area of 253 sq. m. If the width of the path is 4m and the length of the plot is greater than its breadth by 8m, what is the area of the plot? (in m. sq.)

(a)  896

(b)  345

(c)  432

(d)  354

(e)  682

Answer: (b)

Directions (Qs. 2 to 6) : Study the following graph carefully to answer these questions.

2. What is the respective ratio between total number of Blackberry mobile phones sold by stores C, D and G together and the total number of Apple mobile phones sold by the same stores together?

(a)  3 : 4

(b)  2 : 3

(c)  4 : 9

(d)  3 : 5

(e)  5 : 9

Answer: (None)

3. Average number of Nexus mobile phones sold by all the given stores together is by what percent more than the number of BlackBerry mobile phones sold by all the given stores together?

(a) 

(b) 

(c) 

(d) 

(e) 

Answer: (a)

4. What is the difference between number of mobile phones of all three brands sold by stores F and A?

(a)  3,60,000

(b)  3,42,000

(c)  4,20,000

(d)  3,48,000

(e)  3.56.000

Answer: (c)

5. If the price of each BlackBerry mobile is Rs. 22,000, price of each Nexus mobile is Rs. 25,000 and the price of each Apple mobile is Rs. 28,000, by what percent are the total sales of store F more than the total sales of store D?

(rounded off to nearest integer)

(a)  34

(b)  31

(c)  44

(d)  36

(e)  42

Answer: (e)

6. The number of Nexus mobile phones sold by each store increased by 15% from 2008 to 2009, and the number of Apple mobile phones sold by each store increased by 10% from 2008 to 2009. What was the total number of Nexus and Apple mobile phones sold by stores B and E together in 2009?

(a)  12,72,100

(b)  12,79,000

(c)  11,77,200

(d)  12,77,200

(e)  11,72,100

Answer: (b)

Directions (Qs. 7 to 11) : What approximate value will come in place of questions mark (?) in the given questions? (You are not expected to calculate exact value).

7. 92/37.11 = ? – 163.02

(a)  534

(b)  208

(c)  329

(d)  424

(e)  256

Answer: (b)

8. 0032 + 23.982 – (1282.998 + 578.898) + 6.892 = ?

(a)  549

(b)  678

(c)  763

(d)  635

(e)  718

Answer: (None)

9. 99/31.12 + 323.898 ÷ 8.892 = ?

(a)  42

(b)  18

(c)  50

(d)  32

(e)  59

Answer: (c)

10. (√015 – √168.996)2/(√195.989 – √120.996)2 = ?

(a)  97

(b)  58

(c)  81

(d)  72

(e)  61

Answer: (c)

11. (29.989% of 4530.11) – (22.04% of 4599.99) = ? + 125.99

(a)  289

(b)  296

(c)  278

(d)  221

(e)  323

Answer: (d)

12. The speed of the boat in still water is 16 km/h and speed of the current is 2 km/h. It takes a total of 6.5 hours to row upstream from point A to point B and downstream from point B to point C. If the distance from point A to point B is two-thirds the distance between point B and C, what is the total distance travelled by the boat (both upstream and downstream)?

(a)  112 kms

(b)  98 kms

(c)  124 kms

(d)  90 kms

(e)  105 kms

Answer: (e)

13. 24 men can complete a piece of work in 14 days. 2 days after they started working, 4 more men joined them and after 2 more days 6 men left. How many more days will they now take to complete the remaining work?

(a) 

(b) 

(c) 

(d) 

(e) 

Answer: (a)

Directions (Qs. 14 to 18) : Read the following information carefully to answer these questions.

   In college ‘B’, in a group of 1200 students boys and girls are in the ratio of 7 : 5 respectively. Students are studying either Computer Science or IT. Each one of them likes one or  more type of books out of Novel, Biography or Science Fiction.

Out of the boys, 48% study Computer Science and remaining study IT. Out of the boys studying Computer Science, 1/6th like only Novel, 1/4th like only Novel and Biography. 1/6th like only Novel and Science Fiction. 1/4th like only Novel and Science Fiction. 1/4th like all three types of books. Out of the boys studying IT, 1/7th like only Novel, 1/4th like only Novel and Biography, 1/13th like only Novel and Science Fiction. 1/7th like all three types of books.

Out of the girls, 56% study Computer Science and remaining IT. Out of the girls studying Computer Science, 1/7th like only Novel, 1/8th like only Novel and Biography. 1/4th like only Novel and Science Fiction. 1/8th like all three types of books. Out of the girls studying IT, 1/5th like only Novel, 1/11th like only Novel and Biography. 1/4th like Biography. 1/4th like only Novel and Science Fiction. 1/5th like all three types of books.

14. Out of the boys studying Computer Science number of boys who like only Science Fiction is 1/7th of the boys who like Novel. What is the total number of boys studying Computer Science who like either only Novel or only Science Fiction?

(a)  90

(b)  108

(c)  104

(d)  96

(e)  82

Answer: (d)

15. Total number of girls who like only Biography is 75% of the total number of girls who like only Novel. Total how many girls like Science Fiction?

(a)  291

(b)  298

(c)  198

(d)  287

(e)  185

Answer: (b)

16. If the total number of boys studying IT who like Science Fiction is 165, how many boys studying IT like only Biography?

(a)  63

(b)  47

(c)  46

(d)  67

(e)  56

Answer: (e)

17. What percent of the total number of students (boys and girls studying Computer Science and IT) like all three types of books?

(a) 

(b) 

(c) 

(d) 

(e) 

Answer: (d)

18. How many girls studying IT do not like Novel?

(a)  49

(b)  51

(c)  43

(d)  65

(e)  57

Answer: (e)

Directions (Qs. 19 to 24) : Refer to the pie charts answer these questions :

19. The number of Academic Books published by one Company E is what percent less than the number of Non-Academic Books published by the same company?

(a)  25%

(b)  30%

(c)  35%

(d)  22%

(e)  20%

Answer: (a)

20. The number of Non-Academic Books published by one Company C is what percent of the total number of Non-Academic Books published by Companies B and D together?

(a)  36

(b)  40

(c)  38

(d)  48

(e)  32

Answer: (a)

21. What is the respective ratio between the number of Academic Books published by Company A and the number of Non-Academic Books published by Company E?

(a)  5 : 9

(b)  9 : 11

(c)  7 : 9

(d)  4 : 5

(e)  3 : 5

Answer: (a)

22. What is the difference between the number of Academic Books and Non-Academic Books published by Company F?

(a)  580

(b)  640

(c)  512

(d)  768

(e)  456

Answer: (d)

23. The number of Non-Academic Books published by Company A is approximately what percent more than the number of Academic Books published by Company E?

(a)  26

(b)  21

(c)  10

(d)  18

(e)  32

Answer: (b)

24. How many Non-Academic Books were published by Company D?

(a)  3722

(b)  4096

(c)  3264

(d)  3584

(e)  2880

Answer: (b)

25. 1500 were invested for 5 years in scheme A which offers simple interest at a rate of 14% p.a. The amount received after 5 years and some additional money, is then invested in scheme B, for 2 years, which offers compound interest (compounded annually) at a rate of 20% p.a. If the compound interest received from scheme B after 2 years is Rs. 1408, what was the additional money invested in scheme B apart from the amount received from scheme A?

(a)  Rs. 450

(b)  Rs. 650

(c)  Rs. 500

(d)  Rs. 280

(e)  Rs. 520

Answer: (b)

26. How much time will train A take to completely cross train B running in opposite direction (towards each other)?

I. The total length of train A and train B together is 456m.

II. Train B can cross an electric pole in 8 seconds. The respective ratio of speed of train A and train B is 7 : 9.

(a)  The data either in statement I alone or in statement II alone are sufficient to answer the question.

(b)  The data in both the statements I and II together are not sufficient to answer the question.

(c)  The data in both the statements I and II together are necessary to answer the question.

(d)  The data in statement I alone are sufficient to answer the question while the data in statement II alone are not sufficient to answer the question.

(e)  The data in statement II alone are sufficient to answer the question while the data in statement I alone are not sufficient to answer the question.

Answer: (c)

27. How many labourers are employed to do the piece of work?

I. If all the employed labourers report to work, the concerned piece of work can be completed in 36 days.

II. If 15 of the employed labourers are absent (do not report to work), the concerned piece of work can be completed in 56 days.

(a)  The data either in statement I alone or in statement II alone are sufficient to answer the question.

(b)  The data in both the statements I and II together are not sufficient to answer the question.

(c)  The data in both the statements I and II together are necessary to answer the question.

(d)  The data in statement I alone are sufficient to answer the question while the data in statement II alone are not sufficient to answer the question.

(e)  The data in statement II alone are sufficient to answer the question while the data in statement I alone are not sufficient to answer the question.

Answer: (c)

28. What is the sum of money (in Rs.)?

I. The sum when invested in scheme A which offers simple interest @ 7 p.c.p.a., amounts to Rs. 7,752 in two years.

II. The sum when invested in scheme B, which offers compound interest (compounded annually) amounts to Rs. 8,228 in two years and Rs. 9050.80 in three years.

(a)  The data either in statement I alone or in statement II alone are sufficient to answer the question.

(b)  The data in both the statements I and II together are not sufficient to answer the question.

(c)  The data in both the statements I and II together are necessary to answer the question.

(d)  The data in statement I alone are sufficient to answer the question while the data in statement II alone are not sufficient to answer the question.

(e)  The data in statement II alone are sufficient to answer the question while the data in statement I alone are not sufficient to answer the question.

Answer: (a)

29. What was the percentage of discount given while selling the table?

I. The cost price of the table is Rs. 6,000.

II. 16% profit was earned by selling the table for Rs. 6,960. If there were no discount the earned profit would have been 24%.

(a)  The data either in statement I alone or in statement II alone are sufficient to answer the question.

(b)  The data in both the statements I and II together are not sufficient to answer the question.

(c)  The data in both the statements I and II together are necessary to answer the question.

(d)  The data in statement I alone are sufficient to answer the question while the data in statement II alone are not sufficient to answer the question.

(e)  The data in statement II alone are sufficient to answer the question while the data in statement I alone are not sufficient to answer the question.

Answer: (c)

30. What is the volume of the cylinder? (in m3)

I. The height of the cylinder is 6m less than its radius.

II. The difference between the total surface are and curved surface area of the cylinder is 1232 m2.

(a)  The data either in statement I alone or in statement II alone are sufficient to answer the question.

(b)  The data in both the statements I and II together are not sufficient to answer the question.

(c)  The data in both the statements I and II together are necessary to answer the question.

(d)  The data in statement I alone are sufficient to answer the question while the data in statement II alone are not sufficient to answer the question.

(e)  The data in statement II alone are sufficient to answer the question while the data in statement I alone are not sufficient to answer the question.

Answer: (c)

Directions (Qs. 31 to 36) : Study the following table carefully to answer these questions:

31. What percent of the total number of population for States P, R and V together speaks Hindi language?

(a) 

(b) 

(c) 

(d) 

(e) 

Answer: (e)

32. What is the difference (in thousands) between total employed population from states R, S and T together and the total unemployed population from these three states together?

(a)  8500

(b)  9100

(c)  8700

(d)  9300

(e)  9700

Answer: (e)

33. What percent of the population from states Q and V together is in the age group 21-30 years?

(a) 

(b) 

(c) 

(d) 

(e) 

Answer: (a)

34. If the respective ratio of Urban to Rural population among total population of state S is 4 : 3 and that of state T is 3 : 2, what is the total number of rural population (in thousands) in these two states together?

(a)  2800

(b)  2500

(c)  3000

(d)  3450

(e)  3200

Answer: (b)

35. Out of the total employed population in all the given states, 55% are women and out of them 30% women are married. How many employed women (in thousands) are not married?

(a)  7825

(b)  8328

(c)  8365

(d)  9669

(e)  9163

Answer: (e)

36. What percent of the total population from stats Q and S together is unemployed?

(a) 

(b) 

(c) 

(d) 

(e) 

Answer: (c)

37. Anurag’s annual income is Rs. 6,36,000. He spends 22% of h is monthly income on paying bills, 18% on household items, 12% on paying his children’s fees and 4% he donates to a charity. If two-fifth of the remaining amount he invests in mutual funds, what is the amount left with him every month?

(a)  Rs. 17,850

(b)  Rs. 12,162

(c)  Rs. 9,328

(d)  Rs. 13,992

(e)  Rs. 14,650

Answer: (d)

38. The population of a state in the year 2013 was 1,12,926. If the rate of increase was 10% and 20% respectively from the year 2011 to 2012 and 2012 to 2013, what was the population in the year 2011?

(a)  85,550

(b)  86,450

(c)  79,550

(d)  75,550

(e)  76,950

Answer: (a)

Directions (Qs. 39 to 41) : Answer these questions on the following information.

   There are 4 red balls, 4 green balls and 6 blue balls, in a box.

39. If two balls are drawn randomly, what is t he probability that at least one of them is blue?

(a)  6/13

(b)  7/13

(c)  8/13

(d)  10/13

(e)  9/13

Answer: (e)

40. If one ball is drawn randomly, what is the probability that it is either red or blue?

(a)  5/9

(b)  7/9

(c)  4/7

(d)  3/7

(e)  5/7

Answer: (e)

41. If three balls are drawn randomly, what is the probability that one of them is green and the other two are blue?

(a)  15/91

(b)  24/91

(c)  20/91

(d)  10/91

(e)  12/91

Answer: (a)

42. In what respective ratio two varieties of rice costing Rs. 36 per kg and Rs. 60 per kg be mixed, so that by selling the mixture at Rs. 54 per kg 20% profit is earned?

(a)  3 : 2

(b)  7 : 5

(c)  5 : 3

(d)  5 : 2

(e)  4 : 3

Answer: (c)

43. 8 years ago, Jyoti’s age was equal to the Swati’s present age. If the sum of Jyoti’s age 10 years from now and Swati’s age 6 years ago is 88 years, what was Kusum’s age 14 years ago if Kusum is 8 years younger to Swati? (in years)

(a)  22

(b)  14

(c)  25

(d)  24

(e)  16

Answer: (e)

Directions (Qs. 44 to 48) : In these questions, two equations numbered I and II are given. You have to solve both the equations and mark the appropriate answer.

44. I. 2x2 – 7x + 3 = 0

II. 2y2 – 7y + 6 = 0

(a)  x < y

(b)  x > y

(c)  x ≥ y

(d)  x ≤ y

(e)  Relationship between x and y cannot be established.

Answer: (e)

45. I. 4x2 ­+ 16x + 15 = 0

II. 2y2 + 3y + 1 = 0

(a)  x < y

(b)  x > y

(c)  x ≥ y

(d)  x ≤ y

(e)  Relationship between x and y cannot be established.

Answer: (a)

46. I. 9x2 – 45x + 56 = 0

II. 4y2 – 17y + 18 = 0

(a)  x < y

(b)  x > y

(c)  x ≥ y

(d)  x ≤ y

(e)  Relationship between x and y cannot be established.

Answer: (b)

47. I. 2x2 + 11x + 14 = 0

II. 2y2 + 15y + 28 = 0

(a)  x < y

(b)  x > y

(c)  x ≥ y

(d)  x ≤ y

(e)  Relationship between x and y cannot be established.

Answer: (c)

48. I. 6x2 + 11x + 4= 0

II. 4y2 – 7y – 2 = 0

(a)  x < y

(b)  x > y

(c)  x ≥ y

(d)  x ≤ y

(e)  Relationship between x and y cannot be established.

Answer: (a)

49. A started a business with an investment of Rs. 14,000. After 2 months B joins in with 6/7 of the amount that A invested and A withdraws Rs. 4000. After 2 more months, C joins with Rs. 8000 and A again withdraws Rs. 2000. After a year, if C received Rs. 2,656 as his share then what was the total profit? (in Rs.)

(a)  16,954

(b)  11,668

(c)  12,284

(d)  14,326

(e)  13,548

Answer: (c)

50. In a yoga class there were 12 members. Two members left the class and 4 new members joined. If the average age decreased by 4 years and the total age decreased by 2, what is the new average age of the class? (in years)

(a)  22

(b)  27

(c)  23

(d)  28

(e)  18

Answer: (c)

National Insurance Company Ltd.Recruitment of Administrative Officers Examination Held on 12-4-2015 Reasoning Question Paper With Answer Key

National Insurance Company Ltd.Recruitment of Administrative Officers Examination Held on 12-4-2015
National Insurance Company Ltd.Recruitment of Administrative Officers Examination Held on 12-4-2015 Reasoning Question Paper With Answer Key

National Insurance Company Ltd. Recruitment of Administrative Officers Examination

Held on 12-4-2015

Reasoning

1. How far is point D from point S ?

I. Pont R is 5 m to the south of point S. Point J is 5 m to the north of points S. Point D is 2 m to the west of point J. Point K is to the north of point D. The distance between points D and K is 2 m.

II. Point J is 5 m to the south of point D. Point M is 2 m to the east of point J. Point V is to the south of point M. The distance between points V and M is equal to the distance between points D and J. Point S is 2 m to the west of V.

(A)  The data in statement I alone are sufficient to answer the questions, while the data in statement II alone are not sufficient to answer the question

(B)  The data in statement II alone are sufficient to answer the question, while the data in statement I alone are not sufficient to answer the question

(C)  The data either in statement I alone or in statement II alone are sufficient to answer the question.

(D)  The data in both statements I and II together are not sufficient to answer the question.

(E)  The data in both the statements I and II together are necessary to answer the question.

Answer: (C)

2. Among students P, Q, R, S and T, each having a different height, who is the third tallest ?

I. T is shorter than only one person. R is taller than both P and S.

II. P is taller than only one person. R is shorter than both T and Q.

(A)  The data in statement I alone are sufficient to answer the questions, while the data in statement II alone are not sufficient to answer the question

(B)  The data in statement II alone are sufficient to answer the question, while the data in statement I alone are not sufficient to answer the question

(C)  The data either in statement I alone or in statement II alone are sufficient to answer the question.

(D)  The data in both statements I and II together are not sufficient to answer the question.

(E)  The data in both the statements I and II together are necessary to answer the question.

Answer: (C)

3. How is ‘time’ coded in the given code language ? (Note : All the codes are two letter codes only).

I. In the given code language, ‘take your own time’ is coded as ‘sa nv jr pt’ and in the same code language, ‘fix your study time’ is coded as ‘dq sa pt bz’.

II. In the given code language, ‘come by dinner time’ is coded as ‘od es rg sa’ and in the same code language, ‘come home some time’ is coded as ‘sa gk es co’.

(A)  The data in statement I alone are sufficient to answer the questions, while the data in statement II alone are not sufficient to answer the question

(B)  The data in statement II alone are sufficient to answer the question, while the data in statement I alone are not sufficient to answer the question

(C)  The data either in statement I alone or in statement II alone are sufficient to answer the question.

(D)  The data in both statements I and II together are not sufficient to answer the question.

(E)  The data in both the statements I and II together are necessary to answer the question.

Answer: (E)

4. What is the position of B from the right end in a straight line of 8 people ? (Note : All are facing north.)

I. A sits second from the left end of the line. Only three people sit between A and U. B sits third to the right of X. Neither B nor X is an immediate neighbour of U.

II. Z sits at the extreme right end of the line. E sits third to the left of Z B sits exactly between E and R. U sits second to the right of B.

(A)  The data in statement I alone are sufficient to answer the questions, while the data in statement II alone are not sufficient to answer the question

(B)  The data in statement II alone are sufficient to answer the question, while the data in statement I alone are not sufficient to answer the question

(C)  The data either in statement I alone or in statement II alone are sufficient to answer the question.

(D)  The data in both statements I and II together are not sufficient to answer the question.

(E)  The data in both the statements I and II together are necessary to answer the question.

Answer: (C)

5. How is B related to F ?

I. A is the son of Z. B is married to Z. F is the mother of B.

II. F is the wife of W, K and B are the children of W. A is the son of B.

(A)  The data in statement I alone are sufficient to answer the questions, while the data in statement II alone are not sufficient to answer the question

(B)  The data in statement II alone are sufficient to answer the question, while the data in statement I alone are not sufficient to answer the question

(C)  The data either in statement I alone or in statement II alone are sufficient to answer the question.

(D)  The data in both statements I and II together are not sufficient to answer the question.

(E)  The data in both the statements I and II together are necessary to answer the question.

Answer: (D)

Directions- Study the given information and answer the given question.

A consultant hired by company XYZ, concluded the following based upon the last five years’ database of the company.

(a) The less appreciated an employee feels his work is the more, money/reward he wants for it.

(b) An employee becomes more productive on seeing the fruits of his preivous labour.

(c) The harder project an employee accomplishes, the prouder he/she feels of it.

6. Which of the following incidents of company XYZ can not be a basis/source of the consultants conclusions ?

(A)  For the first time, Ms. Verma handled an extremely important project and succeeded in it with flying colours. This made her more tense than happy as now she would be expected to perform equally well every time

(B)  The performance of the production team saw a considerable improvement after the mid-year meeting in which presentations were shown on their previous accomplishments and future plans

(C)  The performance graph of Mr. Garg has become better since he has been shown the pictures of last years’ review when he received the award for ‘most hardworking employee’

(D)  Mr. Rathore successfully dealt with the first multinational project of the company. He had been one of the best performers since he joined the company, but his happiness of success this time was much more than in the previous ones

(E)  Mr. Dixits’s expectations of the yearly incentive amount is higher than other employees even though he is the least productive member of his team

Answer: (E)

7. Which of the following statements, if taken to be definitely true, contradicts the given conclusions of the Consultants ?

(A)  All the employees who demand rewards or a greater salary in company XYZ, are well appreciated high performers of the company

(B)  Even a small ‘good job’ or ‘well done’ is a better reinforcement than monetary rewards

(C)  One’s own previous accomplishments are their best competitors. A person always tries to out-to what he did earlier

(D)  The more difficult/complicated a task is, more focused and motivated is a person to fulfill it successfully

(E)  The employees who are satisfied with the work environment of company XYZ, are least bothered about the money they carry home every month

Answer: (A)

8. Read the given information and answer the question-

As per recent study it has been found that Company A of city X has the lowest level of attrition rate as compared to those of any other company in the city.

“The only reason for this, as I see it, is that Company A asks its prospective employees to sign a bond of minimum two years.” Statement by a local resident.

Which of the following weakens the statements of the resident suggesting that signing a bond with the employees is the only reason for lower attrition rate in the company ?

(A)  Company A is known for being employee friendly and most employees of the company rank very high on the job satisfaction test conducted by the company every year

(B)  Recent search in some major metropolitan cities of the country has suggested that signing a bond with the employee is one of the factors which influence attrition rate of the company

(C)  Now-a-days, many people in city X prefer to finish their education abroad and thereafter seek employment there

Answer: (B)

9. Read the following information carefully and answer the question which follows-

“Although one would like to venture into areas where no competition exists, I would rather set-up my business where higher competition prospects are expected, “Ravi, the owner of a business start-up.

Which of the following statements support/s Ravi’s opinion ?

1. Competition leads to innovation. If one is the only player in the field, it becomes difficult to improve.

2. Chances of finding high efficiency job ready candidates is higher if many companies doing similar businesses exist.

3. Market challenges encourage one to pay greater attention to and concentrate more on target groups where benefits are maximum.

4. Less competition is an indicator of t he fact that existing market is too small to make the desired profits.

(A)  Only 4

(B)  Only 2

(C)  All 1, 2, 3 and 4

(D)  Only 3

(E)  None of these

Answer: (D)

Directions—Study the information carefully and answer the question.

    J, K, L, M, N, O, P and Q are sitting around a circular area at equal distances between each other, but not necessarily in the same order. Some of the people are facing the centre while some face outside (i.e., in a direction opposite to the centre.)

Note-Facing the same direction means if one faces the centre then the other also faces the centre and vice-versa.

O sits second to the left of M. M faces the centre. K sits to the immediate left of O. Only three people sit between K and J. P sits third to the left of J. L sits to the immediate left of N. Immediate neighbours of K face opposite (i.e., if one neighbour faces the centre, the other neighbour faces outside and vice-versa.) N sits third to the right of K. Both N and L face opposite directions. Q faces the same direction as K.

10. Who sits exactly between O and P, when counted from the left of O?

(A)  Q

(B)  K

(C)  L

(D)  N

(E)  J

Answer: (B)

11. Four of the following five are alike based on the given seating arrangement and so form a group. Which is the one that does not belong to that group ?

(A)  P

(B)  J

(C)  Q

(D)  K

(E)  L

Answer: (E)

12. Who sits second to the left of J ?

(A)  K

(B)  L

(C)  M

(D)  P

(E)  Q

Answer: (B)

13. What is K’s position with respect to N ?

(A)  Second to the left

(B)  Immediate left

(C)  Immediate right

(D)  Third to the left

(E)  Third to the right

Answer: (D)

14. How many people in the given arrangement face the centre ?

(A)  Three

(B)  Two

(C)  Four

(D)  Five

(E)  One

Answer: (A)

Directions- Study the information carefully and answer the question.

    When a word and number arrangement machine is given an input line of words and numbers, it arranges them following a particular rule. The following is an illustration of input and rearrangement : (All the numbers are two digit numbers)-

Input : 51 pour 32 start now 23 46 house.

Step I : 23 51 pour 32 start now 46 house.

Step II : 23 start 51 pour 32 now 46 house.

Step III : 23 start 32 51 pour now 46 house.

Step IV : 23 start 32 pour 51 now 46 house.

Step V : 23 start 32 pour 46 51 now house.

Step VI : 23 start 32 pour 46 now 51 house.

   and step VI is the last step of the above arrangement.

As per the rules followed in the above steps, find out in each of the following questions the appropriate step for the given input.

15. Step II of an input is : 18 task bear cold dish 81 63 31.

How many more steps will be required to complete the rearrangement ?

(A)  Three

(B)  Four

(C)  Five

(D)  Six

(E)  None of these

Answer: (C)

16. Input : 72 59 37 go for picnic 24 journey

How many steps will take to complete the rearrangement ?

(A)  Three

(B)  Four

(C)  Five

(D)  Six

(E)  None of these

Answer: (D)

17. Input : nnice flower 34 12 costly height 41 56

Which of the following will be step III ?

(A)  12 nice 34 height flower costly 41 56

(B)  12 nice 34 height 41 flower costly 56

(C)  12 nice 34 flower costly height 41 56

(D)  12 nice flower 34 costly height 41 56

(E)  None of these

Answer: (A)

18. Step III of an input is : 15 yes 29 ask for soap 42 37

Which of the following is definitely the input ?

(A)  ask yes 29 15 for soap 42 37

(B)  yes ask 15 29 for soap 42 37

(C)  29 15 yes ask for soap 42 37

(D)  Cannot be determined

(E)  None of these

Answer: (A)

19. Step II of an input is : 16 victory 19 36 53 store lake town

Which of the following will be step V ?

(A)  16 victory 19 town store 36 53 lake

(B)  16 victory 19 town 36 store 53 lake

(C)  16 victory 19 town 36 53 store lake

(D)  There will be no such step

(E)  None of these

Answer: (D)

20. Statements :

No paper is a file:

Some papers are worksheets.

All worksheets are notebooks.

Conclusion :

I. Some notebooks are papers.

II. No notebook is a paper.

(A)  Neither conclusion I nor II is true

(B)  Both conclusions I and II are true

(C)  Only conclusion I is true

(D)  Only conclusion II is true

(E)  Either conclusion I or II is true

Answer: (E)

21. Statements :

No clue is a puzzle.

All riddles are puzzles.

All questions are clues.

Conclusions :

I. Atleast some puzzles are questions.

II. All clues are questions.

(A)  Neither conclusion I nor II is true

(B)  Both conclusions I and II are true

(C)  Only conclusion I is true

(D)  Only conclusion II is true

(E)  Either conclusion I or II is true

Answer: (A)

22. Statements :

Some controls are steers.

All steers are drives.

No drive is a navigation.

Conclusion :

I. Some drives are definitely not controls.

II. All navigations being controls is a possibility.

(A)  Neither conclusion I nor II is true

(B)  Both conclusions I and II are true

(C)  Only conclusion I is true

(D)  Only conclusion II is true

(E)  Either conclusion I or II is true

Answer: (C)

23. Statements :

Some controls are steers.

All steers are drives.

No drive is a navigation.

Conclusion :

I. Atleast some controls are drives.

II. All drives are steers.

(A)  Neither conclusion I nor II is true

(B)  Both conclusions I and II are true

(C)  Only conclusion I is true

(D)  Only conclusion II is true

(E)  Either conclusion I or II is true

Answer: (C)

24. Statements :

No paper is a file.

Some files are worksheets.

All worksheets are notebooks.

Conclusion :

I. Some worksheets being papers is a possibility.

II. Al files are notebooks.

(A)  Both conclusion I nor II is true

(B)  Both conclusions I and II are true

(C)  Only conclusion I is true

(D)  Only conclusion II  is true

(E)  Either conclusion I or II is true

Answer: (C)

25. Statements :

No clue is a puzzle.

All riddles are puzzles.

All questions are clues.

Conclusion :

I. No question is a puzzle.

II. No riddle is a clue.

(A)  Neither conclusion I nor II is true

(B)  Both conclusions I and II are true

(C)  Only conclusion I is true

(D)  Only conclusion II is true

(E)  Either conclusion I or II is true

Answer: (B)

Directions – Study the following information to answer the given question.

   Seven different people viz, J, K, L, M, N, O and P have to fly to different destinations viz., Delhi, Ahmedabad, Mumbai, Chennai, Pune, Lucknow and Chandigarh but not necessarily in the same order. Each one of them flies on a different day of a week starting from Monday and ending on Sunday of the same week.

• P flies on Wednesday. The one who flies on Friday flies to Chennai.

• Only one person flies between P and J.

• Only two people flying between M and the one flying to Pune. The one who flies to Pune flies after M. M does not have his flight on Monday.

• Only three people have their flights between the people flying to Pune and Chandigarh.

• O flies immediately after the one flying to Mumabi. J does not fly to Mumbai. O does not fly to Pune.

• Only one person has his flight between O and L.

• The person flying to Delhi flies immediately after the person flying to Lucknow.

• K does not fly to Delhi.

26. Who amongst the following flies to Ahmedabad ?

(A)  J

(B)  M

(C)  K

(D)  P

(E)  O

Answer: (C)

27. On which of the following days does the one flying to Lucknow have has flight ?

(A)  Monday

(B)  Thursday

(C)  Tuesday

(D)  Saturday

(E)  Wednesday

Answer: (A)

28. On which of the following days does N have his flight ?

(A)  Sunday

(B)  Thursday

(C)  Monday

(D)  Saturday

(E)  Tuesday

Answer: (E)

29. Who amongst the following has his flight exactly between the days on which O and L have their respective flights ?

(A)  N

(B)  J

(C)  P

(D)  K

(E)  M

Answer: (D)

30. How many person are scheduled to fly between the flight to Mumbai and Pune ?

(A)  None

(B)  Two

(C)  Four

(D)  Three

(E)  One

Answer: (B)

31. Statements :

D ≥ E > P > L < H; R < N < L

Conclusion : I. N > D

II. H > R

(A)  neither conclusion I nor II is true

(B)  both conclusions I and II are true

(C)  only conclusion I is true

(D)  only conclusion II is true

(E)  either conclusion I or II is true

Answer: (D)

32. Statements :

S = H ≤ J ≤ K = L;

H > N < B; D ≥ L

Conclusion : I. N < L

II. K = B

(A)  neither conclusions I nor II is true

(B)  both conclusions I and II are true

(C)  only conclusion I is true

(D)  only conclusion II is true

(E)  either conclusion I or II is true

Answer: (C)

33. Statements :

S = H ≤ J ≤ K = L;

H > N < B; D ≥ L

Conclusion : I. S = D

II. D > S

(A)  neither conclusion I nor II is true

(B)  both conclusion I and II are true

(C)  only conclusion I is true

(D)  only conclusion II is true

(E)  either conclusion I or II is true

Answer: (E)

34. Statements :

B ≥ Z < X ≤ Q; B ≥ Y = L

Conclusion : I. L = Q

II. L > Q

(A)  neither conclusion I nor II is true

(B)  both conclusions I and II are true

(C)  only conclusion I is true

(D)  only conclusion II is true

(E)  either conclusion I or II is true

Answer: (A)

35. Statements :

C > D ≥ F; R < M < F

Conclusion : I. C > M

II. R < D

(A)  neither conclusion I nor II is true

(B)  both conclusions I is true

(C)  only conclusion I is true

(D)  only conclusion II is true

(E)  either conclusion I or II is true

Answer: (B)

Directions-Study the given information and answer the given question.

    Eight people S, T, U, V, W, X, Y and Z live on eight different floors of a building not necessarily in the same order. The lowermost floor of the building is numbered 1, the one above that is numbered 2 and so on till the topmost floor is numbered 8.

Y lives on a odd numbered floor. U lives on a floor that is immediately below Y’s floor. Only four people live between W and S. S lives on one of the floors above W. Z lives on a floor which is immediately above S’s floor. X lives on a floor which is immediately above W’s floor. T does not live on floor numbered 2.

36. Which of the following statements is true according to the given arrangement ?

(A)  None of the given statements is true.

(B)  Only three people live between Y and W.

(C)  Z lives on floor numbered 6.

(D)  T lives on one of the floors below W’s floor.

(E)  Only one person lives above V’s floor

Answer: (A)

37. T lives on which of the following floor numbers ?

(A)  floor number 4

(B)  floor number 1

(C)  floor number 5

(D)  floor number 3

(E)  floor number 6

Answer: (E)

38. Who amongst the following lives on floor number 8 ?

(A)  U

(B)  V

(C)  Z

(D)  S

(E)  X

Answer: (C)

39. Four of the following five are alike in a certain way based on the given arrangement and hence form a group. Which of the following does not belong to that group ?

(A)  YX

(B)  ZT

(C)  VX

(D)  TU

(E)  XU

Answer: (E)

40. Who amongst the following lives immediately below U’s floor ?

(A)  X

(B)  V

(C)  W

(D)  S

(E)  Z

Answer: (A)

© Copyright Entrance India - Engineering and Medical Entrance Exams in India | Website Maintained by Firewall Firm - IT Monteur